SóProvas



Questões de Movimento Retilíneo Uniformemente Variado


ID
346471
Banca
FUNIVERSA
Órgão
SECTEC-GO
Ano
2010
Provas
Disciplina
Física
Assuntos

Foi encontrada, em uma pista asfaltada, uma marca de frenagem retilínea de 31,25 metros de comprimento. Supondo que não tenha havido colisão, e que no término da frenagem o veículo tenha alcançado o repouso, a velocidade do veículo no momento em que acionou os freios, considerando o coeficiente de atrito igual a 0,80 e a gravidade g = 10m/s², era de aproximadamente

Alternativas
Comentários
  • D = V²/250.c               c=coeficiente de atrito

     

    31,25 = V²/250.0,8

    V = 79,05Km/h

     

     

  • Gab D ! basta fazer raiz quadrada de (2*  31,25 * 0,8 * 10) =22,36 m/s multipliiicando por 3,6 obtemos 80Km/h

  • Gabarito: Letra D
     

    1) A distância que um automóvel percorre até parar, após ter  os freios acionados, depende de inúmeros fatores. Essa  distância em metros pode ser calculada aproximadamente  pela expressão D= V²/250.u , onde V é a velocidade em km/h no momento inicial da frenagem e u é um coeficiente adimensional que depende das características dos pneus e do asfalto. 
     

    2) Com isso vamos a resolução:
     

    (FÓRMULA) D = V²/250.u

    D = 31,25m      V² = ?     u = 0,8
     

    - Logo:
     

    31,25 = V²/250.0,8

    31,25 = V²/ 200

    V² = 200.31,25

    V² = 6250 (elevo 6250 ao quadrado e igualo a V², com isso a raiz quadrada de 6250)

    V = 79,06

     

    FORÇA E HONRA.

  • De onde sai essa fórmula?

    D = V² / 250 μ

  • De onde sai esse 250 da fórmula?

    D= V²/250.u

  • a = u.g

    a= 0,8.10

    a=8m/s2

    v2= vo2+2.a.s

    0=vo2+2.8.31,25

    v02=500

    v= 22 x 3,6 = 80 km/h


ID
542251
Banca
CESGRANRIO
Órgão
Transpetro
Ano
2011
Provas
Disciplina
Física
Assuntos

Um corpo de massa 1,0 kg inicia seu movimento com velocidade 2,0 m/s, deslizando sobre uma superfície cujo coeficiente de atrito cinético é 0,1. Após deslocar-se em linha reta de uma distância x, sua velocidade diminui para 1,0 m/s. Quanto à distância x percorrida, está correto afirmar que é igual a
Dado: g = 10 m/s² .

Alternativas
Comentários
  • Fr = Fatc          Fr = m.a    fatc = mi c . N

    m . a = mi c . N           N = P = m . g

    m . a = mi c . m . g        a = mi c . g

    a = 0,1 . 10                   a = 1 newton

    V^2 = Vi^2 + 2.a.x              1^2 = 2^2 + 2.1.x

    1 = 4 + 2x                            -3 = 2x                 

    x = -3/2      ultilizando apenas o módulo do resultado, x = 1,5 

    R: C

  • N = m.g > N = 1.10 > N = 10N

    Fat. = u.N

    Fat. = 0,1 . 10 > 1 N

    Fat. = m.a

    1 = 1.a

    a = 1m/s

    V² = V0² + 2.a.S

    1 = 2² + 2.1.S

    2S = 4 - 1

    S = 3/2

    S = 1,5


ID
568867
Banca
CESGRANRIO
Órgão
Petrobras
Ano
2010
Provas
Disciplina
Física
Assuntos

Considere as informações a seguir para responder à questão.


Uma partícula é lançada verticalmente para cima realizando um movimento retilíneo até atingir o solo. A função horária de posição da partícula é dada por

                                     s(t) = 3,4 + 16t - 5t2


O tempo (t) está medido em segundos e a posição (s), em metros.

Com base nas informações apresentadas acima, analise as afirmativas a seguir.

I – A partícula é inicialmente lançada para cima com velocidade igual a 16 m/s.
II – A partícula atinge sua altura máxima 1,5 segundo após o lançamento para cima.
III – A partícula se move em MRU (Movimento Retilíneo e Uniforme).

É correto APENAS o que se afirma em

Alternativas
Comentários
  • s(t) = 3,4 + 16t - 5t^2

    I) v(t)=s'(t)=16-10t => v(t=0)=16 m/s (C)

    II) hmax=s'(t)=0 => t=1,6s (E)

    III) MRUV (E)

     

  • I) V Basta fazer as equivalências na equação horária de espaço.

    II) F Tempo é 16/10 = 1,6

    III) V Trata-se de MRUV. Desaceleração constante.


ID
568870
Banca
CESGRANRIO
Órgão
Petrobras
Ano
2010
Provas
Disciplina
Física
Assuntos

Considere as informações a seguir para responder à questão.


Uma partícula é lançada verticalmente para cima realizando um movimento retilíneo até atingir o solo. A função horária de posição da partícula é dada por

                                     s(t) = 3,4 + 16t - 5t2


O tempo (t) está medido em segundos e a posição (s), em metros.

A partícula atinge o solo a uma velocidade cujo módulo, em m/s, é

Alternativas
Comentários
  • s(t) = 3,4 + 16t - 5t^2 = 0

    t=-16+-(16^2+20*3.4)^0.5/(-10)=1.6-+(256+68)^0.5/10

    t=1.6-+(324)^0.5/10=1.6+-1.8=3.4 ou -0.2 (foi lançada acima do solo)

    v(t)= 16- 10t=16-10(3.4)=16-34=-18 m/s (o - indica direção)

     

  • A resposta dá aproximada

    Utilizando a equação de Bernoulli

    V^2 = Vo^2 + 2.g.DeltaS

    A partícula foi lançada a 3,6m e retorna nesse ponto com a mesma velocidade inicial = 16m/s

    Na descida g é positivo, logo a partícula atinge o solo com V^2 = 16^2 + 2.10.3,4 = 18,2 m/s

  • s(t) = 3,4 + 16t - 5t2

    S0 = 3,4m

    V0 = 16 m/s

    a = -10 m/s2

     

    Joga em Torricelli:

    V^2 = V0^2 + 2 . a . deltaS

     

    Saca que a equação dada pela questão corresponde ao movimento de subida, por isso a aceleração tá negativa.

    Como a resposta é a velocidade de chegada ao solo, o movimento é de descida, aceleração positiva

     

    V^2 = 16^2 + 2 . 10 . 3,4      (o delta S é Si - Sf, como o Sf está no logo, é igual a 0)

    V^2 = 256 + 68

    V = raiz de 324 = 18 m/s

  • O detalhe é lembrar de considerar So= 3,4m.

    Lançando na equação de Torricelli V= 18m/s.

  • Eu peguei a equação S(t) e resolvi por baskhara, com isso encontrei o valor de t.

    Apos isso, derivei a equação e achei a equação da velocidade em função de t e substitui o valor de t e cheguei na resposta correta!


ID
568987
Banca
CESGRANRIO
Órgão
Petrobras
Ano
2010
Provas
Disciplina
Física
Assuntos

Uma partícula de massa 750 g desloca-se sobre uma retagraduada em metros. Sua posição (em metros) sobre essa reta é dada, em função do tempo, por

s(t) 2t  0,4 t2

estando t em segundos. A variação da quantidade de movimento, em kg.m/s nos 5 primeiros segundos dedeslocamento, vale

Alternativas
Comentários
  • s(t)=2t+0,4t^2

  • Derivando a equação do deslocamento, temos: 

    s'(t) = v(t) = 0,8 t

    Em t=5s:

    v(5) = 0,8 x 5 = 4,0 m/s

    A variação da Quantidade de Movimento será: 

    Q(5) - Q(0)
    m x v(5) - m x v(0)

    0,75 x 4 - 0,75 x 0 = 3,00 kg.m/s (LETRA C)


ID
572680
Banca
Marinha
Órgão
ESCOLA NAVAL
Ano
2009
Provas
Disciplina
Física
Assuntos

Um carro de testes parte do repouso com uma aceleração constante de 6,00m/ s2 em uma pista retilínea. Ao atingir a velocidade de 216km/ h, é submetido a uma desaceleração constante até parar. Qual foi o módulo da desaceleração, em m/ s2, considerando que a distância total percorrida pelo carro foi de 750m?

Alternativas
Comentários
  • Primeiro, deveremos calcular a distancia ate o ultimo momento que o carro ACELEROU.

    obs: espaco inicial = 0 velocidade inicial = 0

    216 km/h = 60 m/s

    formula de torriceli: 60^2 = 0^2 + 2.6.S

    3600 = 12S

    S = 300m

    Ja que o espaco total percorrido foi 750m e 300m foi acelerado, 450m serao retardados.

    Agora é só calcular a desaceleracao.

    0^2 = 60^2 + 2.a.450

    -3600 = 900a

    a = - 3600/900

    a = - 4 m/s^2


ID
685984
Banca
COPESE - UFT
Órgão
UFT
Ano
2010
Provas
Disciplina
Física
Assuntos

Uma pedra, partindo do repouso, cai verticalmente do alto de um prédio cuja altura é “h”. Se ela gasta um segundo (1s) para percorrer a última metade do percurso qual é o valor em metros (m) que melhor representa a altura “h” do prédio?
Desconsidere o atrito com o ar, e considere o módulo da aceleração da gravidade igual a 9,8 m/s 2 .

Alternativas
Comentários
  • Resolução:

    https://brainly.com.br/tarefa/3012


ID
699832
Banca
FUNIVERSA
Órgão
PC-DF
Ano
2012
Provas
Disciplina
Física
Assuntos

Na  questão , caso seja necessário, considere as seguintes informações. 


1) As grandezas vetoriais estão representadas por letras em negrito. Por exemplo, a letra F (em negrito) indica o vetor força, enquanto a letra F (sem negrito) indica o módulo do vetor força.


2) As expressões trigonométricas estão abreviadas da seguinte forma: 


                                                      seno = sen

                                                 cosseno = cos

                                                  tangente = tg 


3) A aceleração da gravidade está representada  por g = 10 m/s2

                             Tempo (h:min)        03:02      03:06         03:11      03:16        03:24

                             Hodômetro (km)   1.583,5   1.586,9      1.594,3    1.598,4     1.615,1 

A velocidade média de um automóvel que se desloca em linha reta (movimento retilíneo), cuja quilometragem e cujo tempo são dados na tabela, é de, aproximadamente,

Alternativas
Comentários
  • GAB: A   !!  Vm = Variação do Espaço/ Variação do Tempo

    Vm= 1615,1-1583,5/03:24-03:22 =  1,43km/min

  • Gabarito: Letra A

    1) Partindo do princípio que Vm = Variação do Espaço(S)/ Variação do Tempo (t)

    2) Vamos agora calcular os tempos e distâncias percorridas:

     

    I - Tempo percorrido

    03:06 - 03:02 = 4 min (t1)

    03:11 - 03:06 = 5 min (t2)

    03:16 - 03:11 = 5 miin (t3)

    03:24 - 03:16 = 8 min (t4)

    II - Distância percorrida

    1586,9 - 1583,5 = 3,4 km (S1)

    1594,3 - 1586,9 = 7,4 Km (S2)

    1598,4 - 1594,3 = 4,1 Km (S3)

    1615,1 - 1598,4 = 16,7 Km (S4)


    3) Agora vamos fazer aplicação da fórmula:
     

    Vm = S1 + S2 + S3 + S4 / t1 + t2 + t3 + t4 

    Vm = 3,4 + 7,4 + 4,1 + 16,7 / 4 + 5 + 5 + 8

    Vm = 31,6 / 22

    Vm 1,43 Km/min (Valor Aproximado)



    FORÇA E HONRA.


ID
699835
Banca
FUNIVERSA
Órgão
PC-DF
Ano
2012
Provas
Disciplina
Física
Assuntos

Na  questão , caso seja necessário, considere as seguintes informações. 


1) As grandezas vetoriais estão representadas por letras em negrito. Por exemplo, a letra F (em negrito) indica o vetor força, enquanto a letra F (sem negrito) indica o módulo do vetor força.


2) As expressões trigonométricas estão abreviadas da seguinte forma: 


                                                      seno = sen

                                                 cosseno = cos

                                                  tangente = tg 


3) A aceleração da gravidade está representada  por g = 10 m/s2

O projeto brasileiro de trem-bala prevê velocidades acima de 300 km/h, mas há quem defenda que a prioridade deveria ser construir trens com velocidade mais baixa, o que tende a torná-los mais baratos. Já o governo paulista iniciou estudos para avaliar a implantação de trens rápidos a partir dos quais poderá haver conexões entre a capital paulista e algumas cidades, como Campinas, São José dos Campos, Sorocaba e Santos. A ideia é usar trens com velocidades entre 160 km/h e 180 km/h. 


                                               Internet: www.folha.uol.com.br  (com adaptações). Acesso em  27/12/2011.


Considere que um futuro trem rápido entre São Paulo e Sorocaba mova-se ao longo de uma seção reta de via com velocidade de 180 km.h-1 , tendo desaceleração de freamento de 2,0 m.s -2 . Nessa situação, considerando que a desaceleração permaneça constante durante a frenagem, a que distância da estação o maquinista deverá frear para que o trem pare na estação?

Alternativas
Comentários
  • equação de Torriceli  V= V02 + 2αΔs  substituindo obtém-se 625m

  • Gabarito: Letra C
     

    1) Vamos extrair os dados da questão:
     

    aceleração = 2m/s

    V = 180km/h - como a questão está pedindo a resposta em metros vamos transformar p/ m/s, com isso vamos dividir 180 por 3,6 = 50m/s

    V (final) = 0

    S = ? (o que a questão nos pede)


    2) Resolução: 
     

    - Para presente questão usaremos a Equação de Torriceli que é:  V² = V0² + 2.a.S

    - Vamos agora a solução da questão:

    50² = 0² + 2.2.S

    2500 = 4S

    S = 625 (Letra D)



    FORÇA E HONRA.

  • faltou se atentar para a desaceleração, que neste caso implica na aceleração negativa. Portanto, a= -2

ID
701305
Banca
CESGRANRIO
Órgão
Petrobras
Ano
2012
Provas
Disciplina
Física
Assuntos

Um corredor percorre, em linha reta, 5.000 m em 30 min. Sabendo que sua velocidade variou entre 8,0 km/h até 14,0 km/h, determine a velocidade média desse corredor em km/h.

Alternativas
Comentários
  • Bom , eu fiz assim . 5000 M converti para km então / 1000 que deu 5 esse é o AS

    30 min converti  para horas então /60 que de 0,5 que é o AT 

    VM= AS/AT 

    VM= 5/0,5 = 10

    Espero que ajude!

  • Questão deu dados em metros e em minutos e pediu em km/h:

     5.000 m em 30 min

    Fazendo a conversão, 5000/1000 = 5km

    30/60 = 0,5h

    Fórmula de velocidade média = S/T 

    5/0,5 = 10

  • 5000m = 5km

    30min = 1/2h

     

    Vm = Δs/Δt

    Vm = 5/ 1/2 (multiplica a primeira pelo inverso da segunda)

    Vm = 5x2

    Vm = 10km/h

     

    Gabarito: B


ID
701308
Banca
CESGRANRIO
Órgão
Petrobras
Ano
2012
Provas
Disciplina
Física
Assuntos

Um carro segue por uma estrada horizontal e retilínea de comprimento L = 8,0 km. Os primeiros 4,0 km são percorridos em 4,0 min. Os últimos 4,0 km são percorridos com uma velocidade V tal que a velocidade média sobre o percurso total é de 30 km/h.

A velocidade V, em km/h, é igual a

Alternativas
Comentários
  • Velocidade média = 30km/h

    Distância total=8km

    Tempo total=?

    Vm=Dt/Tt

    30=8/Tt

    Tt=8/30=>16/60h (ou seja, o tempo total é 16 minutos)

    Como já sabemos que foram gastos 4 minutos nos primeiros 4km, nos outros 4km restantes deverá ser gasto 12 minutos (12/60h ou 0,2h), daí é só aplicar a fómula da velocidade média de novo:

    Vm=4/0,2 => 20km/h (responta letra b)

    Espero ter sido claro

    Avante!

  • A cada 2 min o carro percorreu 1 Km.

    Se foram 8 Km, dá 16 min no total.

    Se consumiu 4 min na primeira parte, sobram 12 min para a segunda parte.

    Para percorrer 4 Km em 12 min é a mesma velocidade que fazer 20 Km em uma hora.

    20 Km por hora.

  • Segundo o enunciado, a variação total do espaço é 8km e a velocidade média do percurso total é 30km/h:

    Olha só, se substituirmos esses dados na equação da velocidade média, conseguiremos encontrar o tempo gasto no percurso total:

    Repare que achamos o Δt em horas, mas precisamos transformá-lo em minutos. Para isso, basta multiplicar por 60:

    Agora nós sabemos o tempo que é gasto para o percurso total e o enunciado nos deu o tempo gasto nos 4 primeiros km, logo, conseguimos descobrir o tempo gasto para percorrer os 4km finais:

    Pronto, temos todos os dados necessários para descobrir V, basta utilizarmos a fórmula da velocidade média:

    Gabarito: D

  • Em três passos:

    ▲S(variação de espaço total) = 8Km

    V(velo. media total) = 30km/h

    ▲t(tempo total do percurso) = ?

    ▲S¹(variação de espaço da 1° metade do percurso) = 4km

    V¹(velo. media 1° percurso) = irrelevante

    ▲t¹(1° metade do percurso) = 4min = 0,06h

    ▲S²(variação de espaço da 2° metade do percurso) = 4km

    V²(velo. media 2 percurso) = ?

    ▲t²(2° metade do percurso) = ?

    1° passo: calcular qual o tempo total do percurso.

    V = ▲S/▲t

    30Km/h = 8Km/▲t

    30.▲t = 8

    ▲t = 8/30 = 0,26 h

    2° passo: calcular o tempo da 2° metade do percurso.

    ▲t = ▲t¹ + ▲t²

    0,26h = 0,06h + ▲t²

    ▲t² = 0,26 - 0,06

    ▲t² = 0,20h

    3° passo: calcular qual a velocidade media do 2° percurso.

    V² = ▲S²/▲t²

    V² = 4Km/0,20h

    V² = 20 Km/h

    Resolver dessa forma pode demorar alguns segundos a mais. Coloquei o máximo de detalhes que pude para que quem tem mais dificuldades, assim como eu tive nesse exercício, compreenda melhor.

    PS: as grandezas não estão elevadas a expoentes, coloquei para identificar cada percurso.

  • t1 =4/60h

    s1=4KM

    t2=?

    S2=4km

    VM( total) = S total / tempo (total)

    30= 8/ 4/60h+t2

    t2=1/5h

    V2= 4/1/5 = 4*5/1=20km/h

  • Gabarito B

    Como trecho foi dividido igualmente (duas metades de 4km) pode usar a expressão:

    Vm = 2 * V1 * V2 = (V1 + V2)

    Descobrindo V1:

    S = 4km

    t1 = 4min = (4/60)h

    V1 = 4km / (4/60)h

    V1 = 4 * 60/4 (Usa propriedade de multiplicar pelo inverso, quando houver uma divisão por fração)

    V1 = 60km/h

    Sabemos que Vm = 30km/h, aplicamos a expressão para encontrar V2:

    30 = 2 * 60 * V2 / (60 + V2)

    30 * (60 + V2) = 120 * V2

    1800 + 30V2 = 120V2

    1800 = 90V2

    V2 = 20km/h

  • Uma pergunta : porque a resposta se baseia somente na V2? Sendo que a questão pergunta de modo genérico a Velocidade?
  • Bom, vamos resolver essa questão da seguinte maneira.

    Primeiro, vamos encontrar o tempo total da distância percorrida.

    1°) Vm total= d/T=

    30 km/h= 8 km/ T=

    T= 8km/30km/h= 0,26h (encontramos o tempo total)

    Agora, vamos encontra a Vm do primeiro percurso.

    A questão nos diz ele percorreu metade do percurso (que é 4km) em 4min, então, primeiramente, vamos transformar "4min" em horas (porque a distância está em Km):

    4min= 0,06h. Então, fica:

    Vm1= 4km/0,06h

    (por aqui já matamos a questão, por que? Porque a questão quer saber só a velocidade média da segunda parte do percurso), Então:

    Vm2= 4km/T (para saber qual é o tempo desse segundo percurso, basta subtrair o Tempo1 do Tempo total, ou seja:

    0,24h

    -

    0,06h

    ---------

    0,20h. Então:

    Vm2= 4km/0,20

    Vm2= 20km/h

    Portanto, a alternativa B é a correta.


ID
715702
Banca
UECE-CEV
Órgão
UECE
Ano
2011
Provas
Disciplina
Física
Assuntos

A velocidade v de um objeto puntiforme que parte com uma velocidade inicial v0 e é submetido a uma aceleração constante a, em cada instante de tempo t, é dada por v = v0 + at. Esta equação pode ser reescrita em termos de uma variável adimensional v’ = v/v0, de modo que v’ = 1 + a’t. Note que v’ é proporcional à velocidade v da partícula. Usando-se o Sistema Internacional de Unidades nas igualdades anteriores, conclui-se que a unidade de medida de a’ é

Alternativas

ID
715726
Banca
UECE-CEV
Órgão
UECE
Ano
2011
Provas
Disciplina
Física
Assuntos

Próximo à superfície da Terra, uma partícula de massa m foi usada nos quatro experimentos descritos a seguir:

1. Foi liberada em queda livre, a partir do repouso, de uma altura de 400 m.

2. Foi submetida a aceleração constante em movimento horizontal, unidimensional, a partir do repouso, e se deslocou 30 m em 2 s.

3. Foi submetida a um movimento circular uniforme em uma trajetória com raio de 20 cm e a uma velocidade tangencial de 2 m/s.

4. Desceu sobre um plano inclinado que faz um ângulo de 60 com a horizontal.

Desprezando-se os atritos nos quatro experimentos, o movimento com maior aceleração é o de número

Alternativas

ID
737488
Banca
Exército
Órgão
EsFCEx
Ano
2010
Provas
Disciplina
Física
Assuntos

Um móvel realiza um movimento cuja trajetória é descrita pela 2 t expressão x (t) = 4 +  4.  A velocidade do móvel, no Sistema Internacional de Unidades, no instante t-2s é:

Alternativas
Comentários
  • Não entendi muito bem como resolver essa questão

  • Pessoal, não sei se estou certo, mais nesta questão não deveria ter sido efetuado o cálculo com

    x= 4 -2²/4

    x= 4 - 4/4

    x= 4 m/s

  • x(t)=4+ 1/4.t^2

    S=S0 +v0+1/2at^2

    V0=0

    S0=4

    a=1/2

    v=v0+at

    v=1/2x2

    v=1m/s

    letra B

    ou deriva a função da posição

    x(t)= 4+1/4t^2

    x'(t)=v(t)

    v(t)= 1/2t

    v(2)=1/2.2

    v(2)=1m/s


ID
799627
Banca
UNICENTRO
Órgão
UNICENTRO
Ano
2011
Provas
Disciplina
Física
Assuntos

Um caminhão passou no quilômetro 100 de uma rodovia com velocidade de 50,0km/h, manteve essa velocidade até o quilômetro 110, quando freou uniformemente e parou em uma placa que indicava 120,0km. No instante em que o caminhão passou no quilômetro 100, uma motocicleta que se encontrava parada nesse local partiu com movimento uniformemente acelerado durante parte do percurso e uniformemente retardado em seguida, até parar no quilômetro 120, chegando junto com o caminhão.

Nessas condições, a velocidade máxima da motocicleta, em km/h, foi, aproximadamente, igual a

Alternativas
Comentários
  • minha resposta foi um chute 

    qual a forma adequada de resolver este problema

  • Simples. 

    Inicialmente foram percorridos 10km à 50km/h.  Isso dá 0,2h para percorrer o primeiro trecho.
    O segundo trecho tem a mesma distância, e foi percorrido em desaceleração constante de 50km/h a 0km/h. Como a velocidade média é a média aritmética, o trecho de 10km foi percorrido à 25km/h. logo durou 0,4h, dando um total de 0,6h nos 2 trechos.

    A bicicleta utilizou o mesmo tempo (0,6h) e a mesma distância (20km) em aceleração e desaceleração constantes. 
    Dividindo 20 por 0,6 temos a velocidade média de 33,33km/h. Como a bicicleta partiu do repouso à velocidade máxima, esta é o dobro da velocidade média. aproximadamente 67km/h.

    Espero que tenha ficado claro. :)
  • 1° momento: Caminhão com velocidade constante:
    Vm=s/t 50=10/t t=10/50 t1=0,2h   2° momento: Caminhão desacelerando   Calculamos a aceleração na eq. de Torricelli: 0=2500+2*10*a -2500=20a a=-2500/20 a=-125km/h   Com a, determinamos o tempo do 2°momento: 0=50-125t t=50/125 t=10/25 t=2/5 t2=0,4h   logo o tempo para o caminhão será t=0,6h   Para a motocicleta.
    O tempo para percorrer os 20 km é o mesmo, t=0,6h.
    sendo assim: vm=20/0,6 = 33,33km/h
    por outro lado a velocidade media pode ser determinada atraves da media entre a velocidade máxima e minima durante  o percurso, logo:
    Vm=(Vmax+Vmin)/2 33,33=(Vmax+0)/2 Vmax=66,66km/h ~ 67km/h

    Letra c
  • Herivelton, entendi tudo até a terceira parte de sua resolução... que fórmula foi essa que você aplicou para calcular o tempo, poderia explicar melhor??? Estou mega confuso

  • Velocidade Média é a média entre duas velocidades, ou seja, Vm=Vmax+Vmin / 2. Se a Vm é 33,3km/h, e a velocidade mínima é 0 (a moto parte do repouso), logo ao substituir na fórmula citada,teremos aproximadamente 67 km/h para a Vmáxima.

  • a terceira parte fica mais clara de ser entendida se utilizarmos da trigonometria: no gráfico velocidade x tempo, a area é igual a distancia percorrida, que sabemos ser 20 km tanto para a motocicleta quanto para o caminhão. Se voce fizer o grafico para a motocicleta verá que formará um triangulo (uma reta ascendente pela aceleração e uma reta descendente pela desaceleração, com o tempo percorrido como base. A velocidade máxima é a altura desse triangulo. logo sabemos que:

    a area do triangulo = distancia = 20km

    area do triangulo = (base x altura) / 2.

    base = tempo total = 0.6 ; altura é a veloc maxima que se procura:

    20 = (0.6 x h) / 2 >>>>>>>>>  h= veloc max= 66,6666666 arredondando 67 gabarito 


ID
863206
Banca
CESPE / CEBRASPE
Órgão
PM-AL
Ano
2012
Provas
Disciplina
Física
Assuntos

Considere que, durante uma perseguição policial, uma viatura conduzida por um oficial combatente tenha atingido 100 km/h em 11,2 s, tendo partido do repouso em um movimento retilíneo uniformemente acelerado. Nessa situação, o módulo da aceleração escalar da viatura, nesse percurso, foi

Alternativas
Comentários
  • v= vo + at
    v=100km/h =27,7 m/s
    vo=0
    a=?
    t=11,2s

    27,7=0+a.11,2
    a= 2,5 m/s^2
  • VM= 100 KM, TRANSFORMA EM METRO 100/3,6= 27,7 m/s

    ACELERAÇÃO
    a=v/t
    a=27,7/11,2
    a= 2, 48 m/s

    RESPOSTA LETRA C
  • Deviam ter botado logo física no edital...

  • Pão, pão, queijo, queijo

    # Conversão de Km/h > m/s

    Km/h [÷3,6 ]

    [...]

    100km/h/3,6

    ≅28

    t ≅ 11

    Aceleração = Velocidade / tempo

    A = 28m/s/11

    A= 2,54.. ≅ 2,55

    1m/s² ≥ 2,55m/s² < 5m/s²

    LETRA C

    APMBB

  • a = DV / Dt


ID
954223
Banca
VUNESP
Órgão
PC-SP
Ano
2013
Provas
Disciplina
Física
Assuntos

Ao ser expelido do cano de 50 cm de comprimento de uma arma em repouso relativamente ao solo, um projétil leva 0,10 s para percorrer, em linha reta e com velocidade constante, a distância de 100 m. Supondo que a massa do projétil seja de 25 g e que seu movimento no interior do cano seja realizado com aceleração constante, a intensidade da força propulsora resultante sobre ele no interior do cano deve ser, em newtons, de

Alternativas
Comentários
  • Alternativa E

    1) Calculando a velocidade da bala após sair da arma:

    Vel = Dist / Tempo = 100m / 0,1s = 1000m/s


    2) Pela equação de Torricelli calcula-se a aceleração dentro do cano da arma:

    (vf)^2 = (vi)^2 + 2.a.d => (10^3)^2 = 0^2 + 2.a.0,5 => a = 10^6 m/s^2


    3) Com isso calcula-se o que se pede na questão que é a Força

    F = m.a = 25.10^-1 . 10^6 =

    .: F = 2,5.10^4 N


  • Alternativa E

    1) Calculando a velocidade da bala após sair da arma:

    Vel = Dist / Tempo = 100m / 0,1s = 1000m/s

    2) Pela equação de Torricelli calcula-se a aceleração dentro do cano da arma:

    (vf)^2 = (vi)^2 + 2.a.d => (10^3)^2 = 0^2 + 2.a.0,5 => a = 10^6 m/s^2

    3) Com isso calcula-se o que se pede na questão que é a Força

    F = m.a = 25.10^-3 . 10^6 = 

    .: F = 2,5.10^4 N

    uma pequena correção no comentário anterior

  • Gabarito E:

    EtapaI, deve-se calcular a velocidade que o projétil sai do cano da arma (para issodesconsidere a distância que ele percorre no interior da arma):

    V=d/t

    V= 100m/0,1s= 1000 m/s2 .

    EtapaII, calcule a aceleração que o projétil sofre no interior da arma:

    Vf2 = Vi2 +2ad

    10002= 0 + 2xax0,5

    Aceleração= 106 m/s2 .

    EtapaIII, calcule o tempo que o projétil percorre no interior da arma:

    Aceleração=var.velocidade/tempo

    Tempo= 1000/1000000

    Tempo= 10-3 s.

    EtapaFinal, calcule a força que o projétil sofre no interior da arma:

    Impulso= Variação daQuantidade de Movimento ( como o projétil parte do repouso, Q inicial = 0)

    Força x tempo= m x Vf

    Força x 10-3 =25x10-3 x 1000

    Força= 25000 = 2,5 x 104 N.


  • Primeiro calcular velocidade (constante - MRU) do projétil fora do cano:

    v=d/t=100/0,1=1000m/s

    Depois calcular aceleração do projétil dentro do cano (MRUV) até atingir velocidade de 1000m/s (calculamos) em 0,10s (dado pela questão), utilizando equação de Toricelli (não entra variável tempo):

    v^2=v0^2+2*a*d

    1000^2=0+2*a*0,5 

    a=1000000=10^6m/s^2

    Por último calcular a força aplicada para fornecer a massa do projétil está aceleração constante, usando a segunda lei de Newton:

    F=m*a=(25/1000)*(10^6)=25000 N=25KN=2,5*10^4N


  • Fiz assim galera:


    Bom eu tenho as seguintes informações:


    m= 25

    dist= 100 m

    t= 0,10 s

    Vo = 0

    V = ?


    1º preciso saber a velocidade final:


    dist = v.t

    100 = v. 0,10

    v = 1000


    2º preciso saber o valor da aceleração:


    V= Vo + a.t

    1000 = 0 + a.0,10

    a= 10.000


    3º aplica a fórmula de N


    Fr = m.a

    Fr= 25 . 10.000

    Fr= 2,5 . 10^4


  • 1) Encontrar a velocidade

    V=ESPAÇO/TEMPO

    V=100m/0,1s

    V=1000 m/s

    2) Encontrar a aceleração dentro do cano da arma

    vf2=vi2 . 2 . a . s

    1000*1000= 0 . 2 . a . 0,5

    1000000*1=a

    a=1000000 ou 10^6

    3) Encontrar a intensidade da força

    F=MASSA . ACELERAÇÃO

    F=0,025 kg . 10^6

    F=2,5*10^-2 . 10^6

    F=2,5*10^4

    GABARITO E


ID
973630
Banca
FGV
Órgão
PC-MA
Ano
2012
Provas
Disciplina
Física
Assuntos

Um motorista afobado, mal aberto o sinal, parte do repouso com uma aceleração constante a. Ao atingir uma velocidade V, percebe à sua frente o próximo sinal fechado.Imediatamente aplica os freios, imprimindo ao carro uma aceleração retardadora constante a', até o repouso.Entre o instante em que partiu do repouso e o instante em que voltou ao repouso, a velocidade escalar média do carro foi:

Alternativas
Comentários
  • Alguém tem a resolução?

  • Alguém tem a resolução?2

  • HAHA treino, treino, treino e uma hora a gente chega lá!!!

    Imagine um veiculo em repouso (V=0 e a=0), partindo do repouso com uma aceleração a, ele alcançará uma velocidade máxima (Vmax) e começará a frear (aceleração negativa), então:

    V escalar media = (Vmax+Vmin)/2

    V escalar media = (V+0)/2

    V escalar media = V/2 m/s

    A aceleração e desaceleração não vão influenciar em nada na velocidade escalar média, afinal o que vai influenciar é a V máxima atingida no trajeto e a minima que é no repouso (V=0).

    LETRA E

  • Para quem prefere resolver através das fórmulas, tenham duas em mente:

    1. Torricelli: V² = Vo² + 2a.ΔS
    2. Equação da velocidade: V = Vo + a.t

    No primeiro momento, o carro parte do REPOUSO (Vo = 0), atinge uma aceleração ''a'', uma velocidade final ''Vf'', percorrendo um ΔS = p em um tempo = x

    Logo, usando Torricelli: Vf = 0² + 2.a.p ---> Vf = 2ap

    Usando Eq. velocidade: Vf = 0 + a.x ---> Vf = ax

    No segundo momento, o carro, tendo atingido a velocidade ''Vf'', que passará a ser a velocidade inicial (Vo), imprime uma desaceleração '' -a' '' (não esquecer o sinal negativo ao fazer as contas), e assim que parar alcançará a velocidade final 0, tendo percorrido um ΔS' = q em um tempo = y

    Logo, usando Torricelli: 0² = Vf² - 2a'.q ---> Vf² = 2a'q

    Usando Eq. velocidade: 0 = Vf - a'y ---> Vf = a'y

    Como velocidade média é Espaço/Tempo, isolamos os respectivos espaços e tempos e fazemos a substituiação:

    Espaço = p + q = vf/2a + vf²/2a'

    Tempo = x + y = Vf/a + Vf/a'

    Ao resolver a equação, obteremos Vf/2, sendo que as acelerações a e a' serão canceladas, significando que independe os valores atribuidos a elas

    1. A questão quer a V med=ΔS/Δt.Sabemos que o tempo é t, e que a area do grafico Vxt nos fornece o espaço percorrido.
    2. A aceleração inicial aumenta a velocidade ate V m/s e depois desacelera até 0 m/s. Isso cria um TRIÂNGULO no gráfico cuja area fornece o espaço percorrido.
    3. ΔS=A(triângulo)=(v.t)/2. Então, V=[(v.t)/2]:t = V/2.
  • No primeiro trecho o carro partiu do repouso:

    Vo = 0

    Então a média da velocidade será:

    Vm = (Vo + V) / 2

    Como Vo = 0, temos que: Vm = V / 2

    E no segundo trecho o carro foi freado até parar:

    V' = 0

    Vm = (V + V') / 2

    Vm = V/2

    Com isso podemos concluir que a velocidade escalar média é a mesma para a ou para a'.


ID
1103680
Banca
UFCG
Órgão
UFCG
Ano
2009
Provas
Disciplina
Física
Assuntos

É dever de todo/a cidadão/ã respeitar as regras de trânsito, a vida própria e a dos outros, o que não faz um motorista alcoolizado à direção. Como exemplo, considere um motorista viajando a 72km/h que observando o sinal vermelho, aplica instantaneamente os freios, e para em 10 segundos, justamente na borda da faixa de pedestres. Suponha que, num outro dia, cometendo a imprudência de consumir bebida alcoólica e dirigir e viajando à mesma velocidade e exatamente na mesma estrada e no mesmo ponto, ele observa a mudança de cor do sinal para o vermelho. Acontece que agora ele demora 0,20 segundo até aplicar os freios. Considerando que o carro freie com a mesma aceleração anterior, pode-se afirmar que avança sobre a faixa de pedestre

Alternativas
Comentários
  • Na primeira situação (sem estar chapado)


    S=So+ VoT+at²/2

    S=0+20.10+2.100/2

    S=200+100

    S=300m (ou seja, em condições normais ele parará o carro em 300 m)


    Na segunda situação (chapado)

    S=So+ VoT+at²/2

    Aumentado o tempo em 0,2 segundo =10,2

    S=20.10,2+2.1104,04/2

    S=200+104,04

    S=304

    Ou seja, 4 metros a mais que na primeira situação




ID
1149556
Banca
IBFC
Órgão
PC-RJ
Ano
2013
Provas
Disciplina
Física
Assuntos

Na análise de um acidente, o perito observou que, em função dos danos provocados no veículo, sua velocidade no momento do impacto era de, no mínimo, 36,0 km/h e que a marca deixada pelos pneus do automóvel, ao serem travados pela frenagem, atingia 15,0 metros. Sabendo que o coeficiente de atrito cinético entre esses pneus travados e o asfalto seco é de 1,8, que não choveu no dia do acidente, que a força de atrito sofre um acréscimo de 10% em função da resistência do ar, esse perito concluiu que, certamente, no início da frenagem, a velocidade aproximada do veículo era de, no mínimo:

Alternativas
Comentários
  • Deve ter um jeito mais simples de fazer, mas é que eu estou penando pra lembrar das coisas aqui (08 anos sem estudar física).

    Primeiro calculamos a Força de Atrito: (vou considerar a letra "c" como coeficiente de atrito)

    Fat = c . P + 10%

    Fat = 1,1 . 1,8 . P

    Fat = 1,1 . 1,8 . m . g

    Fat = 1,1 . 1,8 . m . 9,8 (a questão não falou este dado, mas a gravidade é aproximadamente 9,8)

    Fat = 19,4 . m 

    Sabemos que força igual a massa vezes aceleração, então 

    Fat = m . a = 19,4 . m 

    a = 19,4 (Garda essa informação)


    Depois sabemos que o carro partiu de uma velocidade que queremos descobrir (Vo), colidindo com velocidade (V) de 10m/s2 após frenagem de 15 metros:

    V2 = Vo2 - 2 . a . d

    100 = Vo2 - 2 . a . 15

    100 = Vo2 - 30 . a 

    Vo2 = 100 + 30 . a (Segunda informação)


    Agora é só juntar as duas informações e correr pro abraço:

    Vo2 = 100 + 30 . 19.4

    Vo2 = 100 + 582

    Vo2 = 682


    Vo = 26,11 m/s

    Multiplicando por 3,6, teremos que Vo = 94

    Aproximadamente 95


    p.s.1: se tu utilizar a gravidade como sendo 10, vai achar a resposta mais próxima de 95.

    p.s.2: espero ter ajudado, mas é muito ruim resolver equação aqui.

    p.s.3: quando tem Vo2, quer dizer que é Vo elevado ao quadrado.



  • Outro modo de fazer: Teorema do trabalho-energia

    O trabalho realizado pela força de atrito é: Fat.d .
    O trabalho é igual a variação da energia cinética. Energia cinética é (mv²)/2, onde m é massa e v é velocidade.
    Avariação da energia cinética é: (mv²)/2 - (mv'²)/2 = -Fat.d.1,1  
           (1,1 devido o acrescimo de 10%), v' é velocidade inicial que queremos. O sinal negativo em Fat é devido a força de atrito ser oposta ao deslocamento.
          Como Fat=Peso x Coeficiente, temos Fat=m.g.1,8(g~=10m/s²)
         Reescrevendo:(mv²)/2 - (mv'²)/2 = -m.10.1,8.15.1,1
    Dividindo por m toda a equação:
    v²/2 - v'²/2 = -297
           Substitui-se v=10m/s, dado na questão, de 36km/h transformado em m/s (só foi dividido por 3,6):
    10²/2 - v'²/2 = 297     ;     100/2 - v'²/2 = 297      ;     50 - v'²/2 = 297  
    -v'²/2 = -347    ou      v'²/2 = 347
    v'² = 694                v'= 26,3438...m/s  ( x 3,6 para dar em km/h)           v' = 94,83...km/h

  • Pessoal, fiz de um jeito mais simples do que os acima e cheguei ao resultado. NÃO SEI se a minha dedução foi correta! (Deixo claro hehe)

    V = 36km/h -> 10m/s
    A única força atuante é Fat.

    Mi = 1.8 + 10%1.8 = 2
    Fr = Fat

    Normal = Peso x gravidade

    m.a = Mi . N  -> m.a = Mi. m.g (corta m) a = Mi.g -> a = 2.10 = 20m/s²

    V² = Vo² - 2.a.D (menos pois está desacelerando)
    10² = Vo² - 2.20.15
    Vo² = 700
    Vo = 26.4 m/s -> 95,04km/h

    :)
     


ID
1155592
Banca
CESGRANRIO
Órgão
FMP
Ano
2013
Provas
Disciplina
Física
Assuntos

Atua sobre um objeto uma força resultante constante, conferindo-lhe uma posição, em função do tempo, dada por y(t) = bt3 /2.

Sabendo-se que o tempo é dado em segundos, e a posição, em metros, a constante b tem no SI a dimensão

Alternativas
Comentários
  • y(t) = (b.t³)/2

    [m] = b.[s]³/2

    b = 2 [m]/[s]³

    Como ele só quer a unidade, basta m/s³


ID
1205383
Banca
PUC - RS
Órgão
PUC - RS
Ano
2014
Provas
Disciplina
Física
Assuntos

Muitos acidentes acontecem nas estradas porque o motorista não consegue frear seu carro antes de colidir com o que está à sua frente. Analisando as características técnicas, fornecidas por uma revista especializada, encontra-se a informação de que um determinado carro consegue diminuir sua velocidade, em média, 5,0 m/s a cada segundo. Se a velocidade inicial desse carro for 90,0 km/h (25,0 m/s), a distância necessária para ele conseguir parar será de, aproximadamente,

Alternativas
Comentários
  • Quando o físico não tem tempo, usa torricelli - > Vf²=Vo²+2aD  - > 0²=25² + 2( -5)D  A velocidade final é 0 por que queremos que o carro pare. A aceleração é negativa porque o carro está freiando  - > 0=625 + (-10)D  -> 625 é positivo, então ao passar para o outro lado fica negativo -625 = -10D - > D =-625:-10 = 62,5.   Alguns iniciantes podem achar confuso no começo, mas basta exercitar que tudo fica mais familiar! 


ID
1205503
Banca
FDRH
Órgão
IGP-RS
Ano
2008
Provas
Disciplina
Física
Assuntos

Um automóvel, em eficiência máxima, é capaz de aumentar sua velocidade de 0 a 90 km/h num intervalo de tempo de 12s. Supondo que esse automóvel movimente-se com aceleração constante ao longo de uma pista de corridas retilínea, a distância percorrida por ele para atingir a velocidade final é de, aproximadamente,

Alternativas
Comentários
  • Usando a fórmula...

    S = So + Vot + at²/2 So=0 Vo=0 V = 25 m/s a = dV/dt (Estou usando essa fórmula porque ele afirma que a aceleração é constante) Então: S = 0 + 0 + (V/t).(t²/2) S = Vt/2 S = 25.12/2 = 150 m
  • Gabarito Letra C
     

    1) Primeiramente iremos transformar a velocidade de km/h para m/s:

    90/3,6 = 25m/s

    2) Posteriormente vamos encontrar a aceleração, por meio da definição, e logo após aplicar a equação de Torricelli para determinar o S(variação da distância):

    aceleração (a) = V / t

    a = 25/12 

    a = 2 m/s (valor aproximado)

     

    3) Agora vamos jogar os valores na equação de Torricelli:

    V² = Vo² + 2.a.S

    25² = 0² + 2.2.S

    625 = 4S

    S = 625/4

    S = 156,25


    - Com isso o valor mais próximo a resposta encontrada é a alternativa C (150m).


    FORÇA E HONRA.

  • V0 = 0

    V = 90 km/h => 25 m/s

    TEMPO = 12 s

    S = ?

     

    V = V0 + a.t

    25 = 0 + a.12

    a = 25/12

    a = 2,08 m/s2

     

    S = S0 + V0.T + a.T2/2

    S = 0 + 0.12 + 2,08.12.12/2

    S = 2,08.144/2

    S = 149,76 m ~ 150m

     

     


ID
1205509
Banca
FDRH
Órgão
IGP-RS
Ano
2008
Provas
Disciplina
Física
Assuntos

Um caminhão, cuja massa é de 5,0 toneladas, desloca-se a uma velocidade constante de 54 km/h em uma via retilínea. À sua frente, um semáforo com luz vermelha acionada indica a necessidade de parada do veículo. Para que o movimento de freamento se dê em uma distância de 50 m, a força exercida sobre o caminhão deverá ser de, aproximadamente,

Alternativas
Comentários
  • A força em questão seria a força de atrito---->

    Fat = m.a---->
    V² = Vo² + 2adS ---> V²= 0 --->  Vo² + 2*a*50 = 0 --->
    - (15 m/s)² = 100*a ---> 
    a = - 2,25 m/s² ---> 
    Fat = 5000 kg * 2,25 = 11250 N

  • Velocidade = 54 Km/h / 3,6 = 15 m/s

    Massa = 5 toneladas

    Distância (Delta S) = 50 m

     

    Equação de Torricelli

    V² = V0² + 2 . a . delta S

    15² = 0² + 2 . a . 50

    225 = 100 .a

    a = 225/100

    a = 2,25 m/s²

     

    Segunda Lei de Newton

    F = m . a

    F = 5000 . 2,25

    F = 11, 25 KN

     

    Gabarito Letra C!

  • Eu resolvi pela fórmula do trabalho:

    T = F.d

    Sabendo que a energia envolvida no sistema é igual ao trabalho:. E = T

    E = m.V²/2

    E = 5000.15²/2

    E = 562,5KN

    Logo:

    T = F.d

    562,5 = F.50

    F = 562,5/50

    F = 11,25KN


ID
1216480
Banca
CESGRANRIO
Órgão
Petrobras
Ano
2014
Provas
Disciplina
Física
Assuntos

Um carro se movimenta com velocidade constante de 72 km/h em uma estrada retilínea e horizontal. O motorista, de 70 kg, começa a frear quando avista um sinal de trânsito fechado a 400 m de distância. A aceleração constante imposta ao carro faz com que ele percorra os 400 m e pare no sinal.

Qual é, aproximadamente, em N, o módulo da resultante das forças de interação entre o carro e o motorista durante a frenagem do carro?

Alternativas
Comentários
  • Velocidade inicial = 72km/h = 20m/s

    Vf²=Vo²+2a*(Sf-So)
    0 = 20² + a*2*400
    0 = 400 + 800a
    -400 = 800a
    -400/800 = a
    -0,5 = a

    Fr=m*a
    Fr = 70 * 0,5
    Fr = 35 N


ID
1274812
Banca
CESGRANRIO
Órgão
LIQUIGÁS
Ano
2011
Provas
Disciplina
Física
Assuntos

Um avião está em voo retilíneo horizontal. Ao percorrer uma distância AB em 10 segundos, a sua velocidade aumenta de 350 km/h em A para 422 km/h em B.
No referido trecho, a aceleração média do avião, em m/s2 , é

Alternativas
Comentários
  • 422 -350 = aumento de 72Km/h, dividido por 3,6 = 20m/s ao quadrado

    20m/s em 10 segundos = 2m/s em 1 segundo

  • 20m/s multiplicado po 3,6 e igual a 72km/h mas

     

  • O campo de comentário devia aceitar anexo!


ID
1285804
Banca
CESGRANRIO
Órgão
Petrobras
Ano
2012
Provas
Disciplina
Física
Assuntos

A posição de um corpo em função do tempo é dada por x(t) = 3 + 5t 3 , onde o tempo é medido em segundos, e a posição, em metros.

A aceleração média, em m/s2 , para esse corpo, entre t = 1 e t = 5s, é de

Alternativas
Comentários
  • Gabarito  errado!

    x(1) = 8 m       x(5) = 3 + 5* 5^3 = 628 m            logo :  d = 620 m   para t = 4s

    S = So + Vo*t + a . t^2 /2 = 0 + 0*4 + a .8

    620 = 8 a   e   a = 77,5 m/s

  • Deriva a funcao da posição que vocẽ tem a da velocidade:

    [x(t) = 3 + 5t ³ ]' = 15t² = v(t)

    v(1) = 15

    v(5) = 375

    a = (delta v) / delta t

    a = (375 - 15) / (5-1) => 90

     

  • Acho que ninguém aprende cálculo pra uma prova de nível médio Roberto, mas se tava no edital... hahaha

  • to tentando fazer de várias formas pra não ter que usar a derivada, mas acho que não tem jeito não.

     

    só sai derivando a função posição em relação ao tempo e determinando a velocidade

     

    ainda nao to acreditando que cobraram isso pra nível técnico kkk

  • NO EXEMPLO MOSTRADO PELO SERGIO RIBEIRO, ELE DEDUZIU  QUE Vo=0, OU SEJA, DEDUZIU QUE O MÓVEL ESTA PARTINDO DO REPOUSO.

    NÃO CONSEGUI FAZER DE OUTRA FORMA QUE NÃO FOSSE DERIVANDO A FUNÇÃO FORNECIDA

  • deriva 2 vezes

    3 + 5t³ - POSIÇÃO (t)

    - 15t² - VELOCIDADE (t)

    - 30t - ACELERAÇÃO (t)

    30 . 1 = 30

    30 . 5 = 150

    180 / 2 = 90 m/s²

    @fernandomaringa


ID
1291381
Banca
CESGRANRIO
Órgão
Petrobras
Ano
2012
Provas
Disciplina
Física
Assuntos

Um avião em vôo retilíneo vai do ponto X para o ponto Y em 10 segundos, com aceleração constante de 3 m/s2 .
Se no ponto X sua velocidade é 360 km/h, a distância, em metros, entre os pontos X e Y é

Alternativas
Comentários
  • Convertendo primeiramente a velocidade de km/h para m/s

    360 km/h = 100 m/s

    3,6

     

    S = So + Vo * t + at²

                                 2

    S = 0 + 100 * 10 + 3 * (10)²

                                         2

    S = 0 + 1000 + 150

    S = 1150 m

  • engoliu o 3??


ID
1291384
Banca
CESGRANRIO
Órgão
Petrobras
Ano
2012
Provas
Disciplina
Física
Assuntos

Um avião militar de 10 ton, pousando em um porta-aviões que navega com velocidade constante, é freado por um sistema hidráulico que usa cabos de aço os quais aplicam uma força constante de 300 kN no avião.
Sabendo-se que, no instante em que os cabos engancham no avião, a velocidade relativa entre ele e o porta-aviões é de 278,9 km/h, a distância, em metros, percorrida pelo avião entre o referido instante e o momento em que ele para é

Alternativas
Comentários
  • m = 10 ton = 10000 kg

    F = 300 kN = 300000 N

    V = 278,9 km/h = 77,47 m/s

    Calculando a aceleração:

    F = m . a

    300000 = 10000 . a

    a = 30 m/s^2

    Usando a equação de Torricelli:

    Vf^2 = Vi^2 = 2 . a . Delta S

    0 = 77,47^2 + 2 . 30 .  Delta S

    Delta S = 100, 03 m

    Resposta: Letra d.

  • Fazer essa questão sem calculadora é punk
  • Pesadelo sem calculadora, e a questão "miseráve" ainda não informa que o valor é aproximado... Dá a entender que a gente estaria errando no cálculo trabalhando com esses números quebrados...

    Mas enfim, engole o choro e simbora!!


ID
1361719
Banca
CESGRANRIO
Órgão
Petrobras
Ano
2014
Provas
Disciplina
Física
Assuntos

Considere as informações a seguir para responder a questão.

Uma embarcação, movendo-se em linha reta com velocidade constante de 10 m/s, inicia sua aproximação de
um porto, que se encontra a uma distância de 100 m da embarcação, com desaceleração constante.
Ao chegar ao porto, a velocidade da embarcação é zero.

Qual é o valor da desaceleração, em m/s² , da embarcação?

Alternativas
Comentários
  • V² = Vo² + 2*a* AS
    0² = 10² + 2*a*100
    0=100 + 200a
    200a = -100
    a = -1/2
    a = -0,5

  • Basta aplicar a fórmula de Torricelli: 

    v² = Vo² + 2.a.Δs 

    Como a questão está pedindo a desaceleração, é imprescindível trocar o sinal de (+) para negativo.

    Basta substituir:

    Velocidade = 0;

    Velocidade inicial = 10 m/s;

    Distância = 100 m

    0² = 10² + 2.a.100

    0 = 100 + 200a

    200a = -0 + 100

    200a = 100

    a = 100/200

    a = 0,5 m/s de desaceleração.

    Gabarito letra (E) 

     

  •  

    Esse tipo de questão que não menciona o tempo resolve-se utilizando a equação de Torricelli.

    (Quando o físico quer sair para brincar mas está sem tempo ele usa a equação de torricelli) hahahah

     

    Vf² = Vo² + 2aD - mnemonico -> vovó fudeu vovô mais duas amigas dela

    Vf² = 0 (velocidade final)

    Vo² = 10m/s = (velocidade inicial)

    a = aceleração

    D = 100m (distância percorrida)

     

    Vf² = Vo² + 2aD

    0² = 10² + 2.a.100

    0 = 100 + 200a

    a = 100/200

    a = 0,5m/s² (desaceleração)

     

    Gabarito: E


ID
1381174
Banca
CESGRANRIO
Órgão
Petrobras
Ano
2014
Provas
Disciplina
Física
Assuntos

Um motorista não sabe o caminho para uma cidade. Ele resolve ficar parado na estrada esperando passar um ônibus para aquele destino. Quando finalmente isso acontece, o ônibus passa e mantém velocidade constante de 72,0 km/h. O motorista entra no carro e, 75,0 s depois de o ônibus passar, parte atrás dele com aceleração constante de 2,00 m.s-2 .
Qual é, aproximadamente, em metros, a distância percorrida pelo carro até alcançar o ônibus?

Alternativas
Comentários
  • posição para o ônibus após 75 s:

    s=so+vt

    s=0+20.75

    s=1500 m

    para o carro após 75 s:

    s=so+vot+1/2at^2

    s=0+0+0,5.2.75^2

    s=5625 m

    logo, quando o carro estiver na posição 5625 m, o ônibus estará na posição 3000 m, ou seja, o carro já terá ultrapassado o ônibus.

    Dessa maneira, temos que: 5625 m - 3000 m = 2625 m é o ponto que os dois se encontram.

    Se colocarmos em notação: 2,625.10^3 metros

    No gabarito, de maneira aproximada = 2,5.10^3 metros


  • Após 75s o ônibus estará a 1500ms=so+vts=0+20.75 = 1500m

    Logo,a partir desse ponto e sabendo que a velocidade é 20m/s, a função para a posição do ônibus é:

    s=1500+20t

    O carro parte o repouso e tem aceleração igual a 2m/s², logo, a função que define a posição do carro é:

    s=so+vot+at²/2 (so=0, vo=0 e a=2), por isso

    s=t²
    O ponto que eles se encontrarão será exatamente o ponto em que essas 2 funções possuem a mesma imagem, ou seja, quando as posições (s) forem iguais, logo:

    t²=1500+20t, que é igual a: t²-20t-1500

    resolvendo a equação, encontramos t1=50s e t2=-30s(não existe tempo negativo, por isso t2 está descartada).
    substituindo t1 na função da posição do carro ou do ônibus, descobrimos que dá 2500m, que é igual a letra "c"! Espero ter ajudado!

  • Muito bom o comentário André Mezencio me ajudou muito, a única coisa que deve se corrigido e que ele colocou enganado resposta letra c, mas a resposta que ele chegou foi a D que é a correta.

  • Não entendi o  pq foi utilizado 3000m para a posição do ônibus se encontramos 1500m. 

    ?????


ID
1402666
Banca
PUC-MINAS
Órgão
PUC-MINAS
Ano
2014
Provas
Disciplina
Física
Assuntos

Após certo tempo de queda, a velocidade de um paraquedista torna-se constante. Nessas condições é CORRETO afirmar:

Alternativas
Comentários
  • se é constante, com diz o enunciado, ele vai percorrer distâncias iguais ao tempo.


ID
1451623
Banca
CETRO
Órgão
AEB
Ano
2014
Provas
Disciplina
Física
Assuntos

Um modelo de foguete é lançado a 50 m/s2 por 2s antes da queima total do propelente. O foguete continua subindo. Assumindo a resistência do ar como nula, assinale a alternativa que apresenta a altura máxima que ele alcança.

Obs.: considere g = 10 m/s2 .

Alternativas
Comentários
  • v = v0 + a*t

    v = 100m/s

     

    v² = v0² + 2* a * DeltaS

    0 = 100² - 2 * 10 * DeltaS (menos pois está considerando agora aceleração gravidade, que está freiando o movimento)

    DeltaS = 10000/20 = 500m

  • descobre a velocidade do foguete e substitui na fórmula abaixo:

    ecin=epot

    m.v^2=m.g.h

    letra b

  • GABARITO correto é letra C de acordo com o Cetro.

     

    Primeiro calcula a distância em que o foguete sobe com aceleração.

    velocidade inicial zero, aceleração 50, tempo 2

     

    S=So+Vot +at^2/2

    deltaS= 50 . 2^2   /2

    deltaS= 100

     

    Agora descubra a velocidade final deste movimento.

    v=vo+at

    v=50 . 2

    v= 100

     

    Logo, essa velocidade serve como inicial para o movimento que o foguete faz ate parar no ponto mais alto.

    Lembrando que a velocidade do ponto mais alto é zero e a aceleraçao nesse movimento é a da gravidade sendo contra o movimento de subir então negativa.

     

    v^2=vo^2+2a deltaS

    0 = 100^2+ 2 . (-10) S

    S= 500

     

    Portanto, o foguete subiu acelerando uma distância de 100 metros e andou mais 500 metros até parar.

    500+100 = 600 metros.

     

    Espero ter contribuido com o conhecimento! 

  • Mas eu nao tinha que levar em conta a gravidade ? Entao a aceleraçao não deveria ser 40m/s2 , pois descontando 10 m/s2 da gravidade restaria 40m/s2 ??


ID
1471807
Banca
IDECAN
Órgão
CBM-MG
Ano
2015
Provas
Disciplina
Física
Assuntos

Um veículo mantendo velocidade escalar constante de 72 km/h e em trajetória retilínea se aproxima de um semáforo que se encontra aberto. No instante em que o semáforo se fecha, o veículo passa a apresentar uma desaceleração constante até atingir o repouso, deslocando, nesse trecho de desaceleração, uma distância de 40 m. Considerando que o semáforo se mantém fechado por um minuto, então o intervalo de tempo em que esse veículo fica parado esperando o semáforo abrir é de

Alternativas
Comentários
  • 1º Passo: Converter a velocidade em m/s
    72 / 3,6 = 20m/s


    2º Passo: Achar a aceleração.
    Torricelli: V² = Vo² + 2.a.ΔS

    0² = 20² + 2.a.40
    0 = 400 + 80a
    -80a = 400
    a = 400/-80
    a = -5m/s² (negativo, movimento retardado)


    3º Passo: Achar o tempo de parada.
    Função horária da velocidade: V = Vo + a.t

    0 = 20 + (-5).t
    -20 = -5t
    t = 20/5
    t = 4 segundos


    4º Passo: O semáforo fica vermelho por 60 segundos, então:
    60 - 4 = 56 segundos.

  • De acordo com o enunciado, tem-se:
    vo = 72km/h = 20m/s
    vf = 0
    distância: 40m
    tempo: 1 min = 60s

    vf = vo² + 2a ΔS
    0 = 400 + 2a.40
    0 = 400 + 80a
    -80a = 400
    a = -5m/s² (desaceleração)

    vf = vo + at
    0 = 20 - 5t
    5t = 20
    t = 4s

    Finalmente,
    60s - 4s = 56s

    Resposta D)

  • Dados:

    Vo= 72km/h - 20m/s
    V= 0
    d = 40m
    DT = 1min

    Resoluçåo 

    V² = Vo² + 2.a.ΔS
    0 = 400 + 2.a.40
    a= 400/80 = 5 (negativa por causa da desaceleração)

    V= Vo + a.t
    0= 20 + 5.t
    t= 4s

    1 minuto que ficou parado no trânsito = 60 segundos
    => 60segundos - 4 segundos = 56segundos parado esperando.

     

  • muito fácil 

  • Bizu: Um veículo mantendo velocidade escalar constante de 72 km/h e em trajetória retilínea. (De cara você já sabe que é MRUV)

    Fórmulas:

    vf= vo + a.t
    d= vo.t + a.t²/2
    vf²= vo²+ 2.a.d

  • 72km/h= 20m/s

    V²=Vo²+2aΔs

    0²= 20²+2.a.40

    400=80a

    a= 5m/s²

    [...]

    5= 20/x

    5x= 20

    x= 4segundos de desaceleração

    60minutos - 4 segundos = 56seg

    LETRA D

    APMBB


ID
1518718
Banca
Marinha
Órgão
EAM
Ano
2015
Provas
Disciplina
Física
Assuntos

A posição de uma partícula em Movimento Retilíneo Uniforme varia de acordo com a equação horária [ S = 20 - 4.t ] em unidades do Sistema Internacional. A partir desta equação, após quanto tempo de movimento a partícula passa pela origem dos espaços?

Alternativas
Comentários
  • Olá, boa tarde!

    A posição de uma partícula em Movimento Retilíneo Uniforme varia de acordo com a equação horária [ S = 20 - 4.t ] em unidades do Sistema Internacional. A partir desta equação, após quanto tempo de movimento a partícula passa pela origem dos espaços? Resposta: 5 s.

    Questão de aplicação direta de fórmula. Primeiramente, perceba que se trata de MRU, em que a trajetória do móvel se faz em uma reta (não há curvas) e sua velocidade se mantém constante com o passar do tempo.

    Para o MRU, temos a fórmula da velocidade (v) = delta S / delta T, ou seja, a velocidade é calculada pela divisão da variação do espaço (espaço final - espaço inicial) pela variação do tempo (tempo final - tempo inicial).

    Derivando da anterior, temos a fórmula da posição pelo tempo, que é a usada nessa questão, e nos permite calcular a posição do móvel em MRU a qualquer tempo, já que a velocidade é constante: S final = S inicial + V.T; em que S é o espaço final, V é a velocidade constante e T é o tempo.

    Portanto, com a equação horária S = 20 - 4.t dada pela questão, e sabendo que a "origem dos espaços" corresponde a S=0, basta substituir:

    0=20 - 4.t

    -20 = -4t

    t= 5 s.

    Lembre-se de que no SI (Sistema Internacional), V é em m/s; S em m; a (aceleração) em m/s.s (metros / segundo ao quadrado).

    Muito obrigada, Natália.


  • A questão nos dá a fórmula: S=20-4.T (S=So+v.t). Ela pede a origem, vamos substituir S por 0, logo

    0=20-4.t, vamos passar o negativo para o outro lado, positivando-o.

    4t=20, vamos, agora, dividir!

    t=20/4

    t=5.

    Resposta: B de Brasil, viva as forças armadas!

  • Bizu, sempre que pedir ponto de origem em MUV

    Iguale a 0 a equação, ou seja, S(posição final)=0

  •  S = 20 - 4.t

    0 = 20 - 4.t (0 pois é a posição inicial)

    20 = 4.t

    20/4 = t

    5 = t


ID
1601983
Banca
PUC - GO
Órgão
PUC-GO
Ano
2015
Provas
Disciplina
Física
Assuntos

TEXTO 3  

                                       O outro

     Ele me olhou como se estivesse descobrindo o mundo. Me olhou e reolhou em fração de segundo. Só vi isso porque estava olhando-o na mesma sintonia. A singularização do olhar. Tentei disfarçar virando o pescoço para a direita e para a esquerda, como se estivesse fazendo um exercício, e numa dessas viradas olhei rapidamente para ele no volante. Ele me olhava e volveu rapidamente os olhos, fingindo estar tirando um cisco da camisa. Era um ser de meia idade, os cabelos com alguns fios grisalhos, postura de gente séria, camisa branca, um cidadão comum que jamais flertaria com outra pessoa no trânsito. E assim, enquanto o semáforo estava no vermelho para nós, ficou esse jogo de olhares que não queriam se fixar, mas observar o outro espécime que nada tinha de diferente e ao mesmo tempo tinha tudo de diferente. Ele era o outro e isso era tudo. É como se, na igualdade de milhares de humanos, de repente, o ser se redescobrisse num outro espécime. Quando o semáforo ficou verde, nós nos olhamos e acionamos os motores. 

                                              (GONÇALVES, Aguinaldo. Das estampas. São Paulo: Nankin, 2013. p. 130.)

   No Texto 3, temos referência a semáforo, instrumento usado para controlar o trânsito em cruzamentos de vias. Considere que um carro se move a 54 km/h e está a 31,5 m de um semáforo, quando a luz desse semáforo fica vermelha. O motorista imediatamente aciona os freios, imprimindo uma desaceleração constante ao veículo. A pista está molhada, e o motorista não consegue parar o carro antes do semáforo, passando por ele, ainda vermelho, 3 segundos após o início da freada. Analise os itens que se seguem:


I- A desaceleração do carro durante a freada tem um módulo de 3 m/s2 .

II- O módulo da velocidade do carro no instante em que passa pelo semáforo é de 21,6 km/h.

III- Para conseguir parar o carro no local onde está o semáforo, o motorista deveria imprimir uma desaceleração constante com módulo de 5,5 m/s2 .

IV- Para conseguir parar o carro no local onde está o semáforo com uma desaceleração constante, o motorista levaria um tempo menor que 3 segundos.


Marque a única alternativa que contém todos os itens corretos:


Alternativas

ID
1602061
Banca
PUC - GO
Órgão
PUC-GO
Ano
2015
Provas
Disciplina
Física
Assuntos

TEXTO 7

                                      Ao mar

    Choveu dias e depois amanheceu. Joel chegou à janela e olhou o quintal: estava tudo inundado! Joel vestiu-se rapidamente, disse adeus à mãe, embarcou numa tábua e pôs-se a remar. Hasteou no mastro uma bandeira com a estrela de David... 

     O barco navegava mansamente. As noites se sucediam, estreladas. No cesto de gávea Joel vigiava e pensava em todos os esplêndidos aventureiros: Krishna, o faquir que ficou cento e dez dias comendo cascas de ovo; Mac-Dougal, o inglês que escalou o Itatiaia com uma das mãos amarradas às costas; Fred, que foi lançado num barril ao golfo do México e recolhido um ano depois na ilha da Pintada. Moma, irmão de sangue de um chefe comanche; Demócrito que dançava charleston sobre fios de alta tensão... 

    — A la mar! A la mar! – gritava Joel entoando cânticos ancestrais. Despertando pela manhã, alimentava-se de peixes exóticos; escrevia no diário de bordo e ficava a contemplar as ilhas. Os nativos viam-no passar – um ser taciturno, distante, nas águas, distante do céu. Certa vez – uma tempestade! Durou sete horas. Mas não o venceu, não o venceu! 

     E os monstros? Que dizer deles, se nunca ninguém os viu? 

     Joel remava afanosamente; às vezes, parava só para comer e escrever no diário de bordo. Um dia, disse em voz alta: "Mar, animal rumorejante!" Achou bonita esta frase; até anotou no diário. Depois, nunca mais falou. 

     À noite, Joel sonhava com barcos e mares, e ares e céus, e ventos e prantos, e rostos escuros, monstros soturnos. Que dizer destes monstros, se nunca ninguém os viu? 

     — Joel, vem almoçar! – gritava a mãe. Joel viajava ao largo; perto da África. 

(SCLIAR, Moacyr. Melhores contos. Seleção de Regina Zilbermann. São Paulo: Global, 2003. p. 105/106.)


     No Texto 7 há uma menção a chuva. Suponha que uma gota de chuva, inicialmente em repouso, se forme 2000 m acima da superfície terrestre (altura suficiente para que as gotas atinjam a velocidade terminal). Considerando-se que a força devida ao atrito viscoso (resistência do ar) sobre um objeto seja diretamente proporcional ao quadrado da velocidade e dependa somente dela, e que, para a gota em questão, a constante de proporcionalidade C é igual a 2,0 × 10-6 kg/m, adotando-se a aceleração da gravidade local como 10 m/s2 e convencionando-se que todas as gotas envolvidas partam do repouso, é possível afirmar que:

 I- Se a massa da gota for igual a 1,5 × 10-5 kg, o módulo da velocidade terminal da gota será algo entre 31 km/h e 32 km/h.

II- Se se substituir a gota de água por uma de mercúrio com as mesmas dimensões e formato, mas com massa igual a 2,0 × 10-4 kg, sua velocidade terminal terá um módulo entre 113 km/h e 114 km/h.

III- Se a resistência do ar for desprezada, a gota de água atingirá o solo com velocidade de módulo igual a 720 km/h.

IV- Se a resistência do ar for desprezada e a gota de mercúrio partir da mesma altura e velocidade que a gota de água, por ser mais pesada, ela atingiria a superfície com uma velocidade de módulo consideravelmente maior que a da água.


Com base nas sentenças anteriores, marque a alternativa em que todos os itens estão corretos:


Alternativas

ID
1614016
Banca
CESGRANRIO
Órgão
PUC - RJ
Ano
2013
Provas
Disciplina
Física
Assuntos

Um bloco, a uma altura 2,7 m do solo, escorrega a partir do repouso por uma rampa até chegar à uma superfície horizontal, por onde segue. Não existe atrito entre o bloco e a rampa. O coeficiente de atrito cinético entre o bloco e a superfície horizontal é 0,30.

Calcule a distância em metros que o bloco percorre sobre a superfície horizontal até parar.

Alternativas
Comentários
  • energia mecânica em A de altura h=2,7m e onde v=0 --- E=mV/2 + mgh=m.0/2 + m.10.2,7 --- E=27m --- energia mecânica em B onde h=0 e a velocidade do bloco é V --- E= mV/2 + mgh=mV/2 + m.g.o --- E= mV/2.

     Ema= Emb--- 27m=mV/2 --- V=√54 m/s.

    F=ma --- F=μN Fat=0,3.m.10=3m --- Fr=Fat --- ma=3m --- a=3m/s.

    V = V + 2.a.d --- 0 = (√54) + 2.(-3).d --- d=54/6 --- d=9m

  • Como não existem forças dissipativas na rampa, toda a energia potencial gravitacional irá se transformar em energia cinética no final da rampa.

    Epg = Ec

    m.g.h = m.v²/2

    10.2,7 = v²/2

    v² = 54

    Analisando as forças que atuam sobre o corpo quando ele está na superfície horizontal, veremos que a força resultante é a força de atrito.

    Fat = m.a

    m.10.0,3 = m.a

    a = 3 m/s²

    Como está contra o movimento, a aceleração será negativa.

    a = - 3 m/s²

    Agora aplicaremos Torricelli para encontrar o deslocamento.

    v² = vo² + 2.a.ΔS

    0 = 54 + 2.(-3).ΔS

    6ΔS = 54

    ΔS = 9 m

    GABARITO: LETRA D

  • TEOREMA DA ENERGIA CINÉTICA: TRABALHO DO PESO - TRABALHO DA FAT= VARIAÇÃO DA EC.


ID
1614469
Banca
PUC - RJ
Órgão
PUC - RJ
Ano
2014
Provas
Disciplina
Física
Assuntos

Um carro, deslocando-se em uma pista horizontal à velocidade de 72 km/h, freia bruscamente e trava por completo suas rodas. Nessa condição, o coeficiente de atrito das rodas com o solo é 0,8.

A que distância do ponto inicial de frenagem o carro para por completo?

Considere: g = 10 m/s2

Alternativas
Comentários
  • GABARITO LETRA B

     

    Dados da questão:

    µ=0,8

    Vo= 72km/h   ===> Dividindo por 3,6 para passar m/s === > 20m/s

    g=10 m/s² ===> Dado pela questão

    Vf = 0 m/s  ===> Pois após a frenagem o carro encontra-se parado

    Resolução da questão:

    Isolando as forças no carro , podemos observar no eixo vertical que a normal é igual ao peso ( N = P ) e no eixo horizontal a força de atrito é igual a força resultante. Sendo assim , temos:

    Fat= m.a    ==> µ.N=m.a    ==> 0,8 . P = m.a ===> 0,8.m.g = m.a ===> cortando a massa ==> 0,8.10=a ===> a= 8 m/s²

    Achar o deslocamento que é pedido na questão :

    V²=Vo²+2aΔS ===> 0=20² +2.8.ΔS ===> 16ΔS+400=0 ===> ΔS= -400/16 ===> ΔS = -25m ==> Como não existe deslocamento negativo ==> ΔS =25m

     

  • Em(antes) = Em(depois)                                                72km/h = 20 m/s (basta dividir por 3,6 para fazer essa conversão)

    MV²/2 = FAT*deslocamento

    MV²/2 = M*G*Ue* deslocamento

    V²/2 = G*Ue* deslocamento

    20²/2 = 10 * 0,8 * deslocamento

    deslocamento = 25 metros

  • Velocidade:

    72km/h => 20m/s


    Desaceleração:

    a = µ.g

    a = 0,8.10

    a = 8m/s²


    Encontrar o deslocamento

    V² = Vo² + 2aΔS

    0² = 400 + 2.8.ΔS

    0 = 400 + 16ΔS

    -16ΔS = 400

    ΔS = 400/16

    ΔS 25m


ID
1629262
Banca
Marinha
Órgão
EFOMM
Ano
2015
Provas
Disciplina
Física
Assuntos

Um automóvel, partindo do repouso, pode acelerar a 2,0 m/s2 e desacelerar a 3,0 m/s2 . O intervalo de tempo mínimo, em segundos, que ele leva para percorrer uma distância de 375 m, retornando ao repouso, é de

Alternativas
Comentários
  • Sabemos que em um gráfico que relaciona velocidade e tempo, se calcularmos sua área obteremos o espaço (distância). Como a aceleração é constante, a velocidade tende a aumentar enquanto estiver sendo imprimida aceleração, dessa forma, concluímos que o gráfico será uma reta linear para velocidade, que chegará ao máximo e de acordo com o exercício em um dado momento começa a desacelerar. Temos os dados:


    a= 2m/s²                                espaço percorrido= 375m

    desaceleração= 3m/s²          tempo mínimo para percorre o espaço= ?


    Como o gráfico será um reta inclinada, podemos calcular a área da seguinte forma:


    A=b*h/2


    Interpretando isso para os dados do exercício:


    A= espaço (375m)

    b= tempo (t)

    h=  Vmax


    375=t*Vmax/2


    Sabemos que o deslocamento ΔS é obtido pela soma do deslocamento durante o período de aceleração (Sa) e do deslocamento durante a desaceleração (Sb). ΔS= Sa+Sb.


    V0=0m/s   p/ Sa                    V= 0m/s  p/ Sb               ΔS= 375m


    Usando a seguinte equação: V²= V0²+2*a*Sa ------->  V²=V0²+2*2*Sa ------> Sa= V²/4


    0=V0²+2*3*Sb  ------> Sb= V0²/6


    375=V²/4 + V0²/6


    Fazendo as contas obteremos Vmax= 30m/s


    Substituindo Vmax:              375=t*30/2  ----------->   t= 25s



  • De acordo com o enunciado, tem-se que a área do gráfico V x t fornece o valor da distância percorrida.


    Sendo assim,
    375 = t x Vmáx / 2
    S1 + S2 = 375

    Na aceleração:
    Vfinal² = Vinical² + 2a x S1
    Vmáx² =  0 + 2 x 2 x S1
    Vmáx² = 4S1
    S1 = Vmáx² / 4

    Na desaceleração:
    Vfinal² = Vinical² + 2a x S2
    0 = Vmáx² - 2 x 3 x S2
    Vmáx² = 6S2
    S2 = Vmáx² / 6

    Substituindo os valores, tem-se:
    Vmáx²/4 + Vmáx²/6 = 375
    6Vmáx² + 4Vmáx² = 9000
    10Vmáx² = 9000
    Vmáx² = 900
    Vmáx = 30 m/s

    Finalizando,
    375 = t x Vmáx/2
    375 = t x 30/2
    375 = t x 15
    t = 25 s

    Resposta B)


  • A questão peca muito na interpretação de texto 

  • https://www.youtube.com/watch?v=sG2jZj69A0A Resolução

  • https://www.youtube.com/watch?v=NQkeqDNbALc

    resolução


ID
1637728
Banca
Aeronáutica
Órgão
ITA
Ano
2010
Provas
Disciplina
Física
Assuntos

Caso necessário, use os seguintes dados:
Aceleração da gravidade = 10 m /s²  
Velocidade de som no ar = 340 m /s 
Densidade da água = 1,0 g/cm ³
Comprimento de onda médio da luz = 570 nm

Um corpo de massa M, inicialmente em repouso, é erguido por uma corda de massa desprezível até uma altura H, onde fica novamente em repouso. Considere que a maior tração que a corda pode suportar tenha módulo igual a n Mg. em que n > 1, Qual deve ser o menor tempo possível para ser feito o erguimento desse corpo?

Alternativas

ID
1678600
Banca
Marinha
Órgão
Quadro Complementar
Ano
2014
Provas
Disciplina
Física
Assuntos

o movimento curvilíneo de uma partícula é definido por vx = 25 - 8t e y = 72 - 2t2 , onde vx está em metros por segundo, y em metros, e t em segundos. Sabe-se que, para t=0, x=0. Sendo assim, quais são, respectivamente, a velocidade e a aceleração da partícula quando y=0?

Alternativas

ID
1717078
Banca
Marinha
Órgão
Quadro Complementar
Ano
2015
Provas
Disciplina
Física
Assuntos

A posição de um ponto material que se desloca em linha reta é definida pela relação x = t3-6t2 -15t + 40, onde x é expresso em metros e t em segundos, com t > 0. Determine o instante em que o móvel inverte seu sentido do movimento, e assinale a opção correta.

Alternativas
Comentários
  • o único que deu x positivo foi o t=1 segundo... o restante todos deram números negativos..

     

    se alguém conseguiu entender por gentileza me explique por favor

  •  

    x = t3-6t2 -15t + 40

    Para encontrar o ponto de mudança basta derivar e igualar a zero (encontrar os pontos onde a derivada primeira é zero).

     

    3t²-12t-15 = 0 (dividindo tudo por 3)
    t²-4t-5 = 0

    t1 = -1

    t2 = 5

    Como t >0, a resposta é 5 segundos.


ID
1736875
Banca
FUNCAB
Órgão
CBM-AC
Ano
2015
Provas
Disciplina
Física
Assuntos

Um carro a 120 km/h é freado uniformemente com aceleração de 5 m/s² (em módulo) até parar completamente.A distância percorrida por esse carro até sua parada é de, aproximadamente:

Alternativas
Comentários
  • Equação de Torricelli

  • V² = Vo² + 2aD

    120km/h = 33.3m/s

    0² = 33.3² - 2*5*D

    D = 111m

    Gabarito: letra E

  • Velocidade = 120 => 120/3,6 = 33,33m/s

    Aceleração = -5m/s (negativo pois o carro está freando)

     

    Equação de Torricelli

    V² = Vo² + 2aΔs 

    0² = 33,33² + 2 . (-5) . Δs

    0² = 1.110 - 10Δs

    10Δs = 1.110

    Δs = 1110/10

    Δ​s = 111 m


ID
1774216
Banca
NC-UFPR
Órgão
UFPR
Ano
2015
Provas
Disciplina
Física
Assuntos

Um sistema amplamente utilizado para determinar a velocidade de veículos – muitas vezes, chamado erroneamente de “radar" – possui dois sensores constituídos por laços de fios condutores embutidos no asfalto. Cada um dos laços corresponde a uma bobina. Quando o veículo passa pelo primeiro laço, a indutância da bobina é alterada e é detectada a passagem do veículo por essa bobina. Nesse momento, é acionada a contagem de tempo, que é interrompida quando da passagem do veículo pela segunda bobina.

Com base nesse sistema, considere a seguinte situação: em uma determinada via, cuja velocidade limite é 60 km/h, a distância entre as bobinas é de 3,0 m. Ao passar um veículo por esse “radar", foi registrado um intervalo de tempo de passagem entre as duas bobinas de 200 ms. Assinale a alternativa que apresenta a velocidade determinada pelo sistema quando da passagem do veículo.

Alternativas
Comentários
  • Questão boa p PRF

    --

    Convertendo as unidades p km/h

    3m = 0.003km       

    200ms = 200x10^-3 s = (200x10^-3)/3600 horas

    --

    Logo,     Vm = ΔS/Δt

    Vm = 0,003 / [(200x10^-3)/3600]

    Vm= 54 km/h

  • 200 ms = 200 x 10^3 = 0,2 s = 2/10 = 1/5

    V = d/t

    V = 3/1/5

    V = 3/2 x 5/1

    V = 15 m/s

    V = 15 m/s x 3,6 = 54 km/h

    Resposta: C


ID
1846564
Banca
CONSULPLAN
Órgão
CBM-PA
Ano
2016
Provas
Disciplina
Física
Assuntos

Um automóvel se deslocando em uma trajetória retilínea com velocidade constante de 108 km/h passa a desacelerar 2,5 m/s2 ao se encontrar a uma distância de 100 m de uma ponte e ao entrar na mesma mantém sua velocidade constante até atravessá‐la totalmente. Quanto tempo o automóvel gastou para atravessar a ponte que tem 200 m?

Alternativas
Comentários
  • V² = V0² -2.a.d  (Torricelli)

    V² = 30² - 2.2,5.100

    v=20m/s

     

    V=d/t

    20=200/t

    t=10s

     

    ​Espero ter ajudado!

  • V= 108km/h OU 30m/s

    a= -2,5m/s² (Carro está desacelerando)

    So = 100m

    S = 300m (a ponte com seus 200m e o corpo está a 100m, totatlizando então 300m). 

    300 = 100 + 30t - 2,5t²                                                                                                                                                                                                                      2

    0 = -50 + 30t - 2,5t²

    x = -30 +- V900 - 500                                                                                                                                                                                                           -100

    x' = 0,1minutos

    x''= 0,5minutos

    Resp: Letra C: 10s

  • Tem que entender e interpretar a questão...


ID
1849462
Banca
Exército
Órgão
EsPCEx
Ano
2015
Provas
Disciplina
Física
Assuntos

Um móvel descreve um movimento retilíneo uniformemente acelerado. Ele parte da posição inicial igual a 40 m com uma velocidade de 30 m/s, no sentido contrário à orientação positiva da trajetória, e a sua aceleração é de 10 m/s2 no sentido positivo da trajetória. A posição do móvel no instante 4s é

Alternativas
Comentários
  • De acordo com o enunciado, tem-se:
    S(t) = So + Vo t + at²/2
    S(4) = 40 - 30 . 4 + 10.16/2
    S(4) = 40 - 120 + 160/2
    S(4) = - 80 + 80
    S(4) = 0

    Resposta A)
  • S = So + Vo*t + a*t²/2

     

    S = 40 - 30*4 + 10*4²/2

    S = 40 - 120 + 80

    S = 120 - 120

    S = 0

     

    Gabarito: letra A

  • Questão que se resolve com a fórmula da Função Horária da Posição:

    S(t) = So + Vot + at²/2 (sentado, sozinho, vendo TV até as duas e meia)

     

    So = Posição Inicial = 40m

    v = velocidade = -30 (negativa pois a questão fala que está no sentido contrário)

    a = aceleração = 10m/s²

     

    Resolução:

    S(t) = So + Vot + at²/2     

    S(t) = 40 + (-30).4 + 10.4²/2

    S(t) = 40 - 120 + 10.16/2

    S(t) = -80 + 160/2

    S(t) = -80 + 80

    S(t) = 0

  • A aceleração não deveria ser negativa, já que o móvel é acelerado?
  • Pensei a mesma coisa Mateus, por isso errei kkk

  • Eu também não entendi por que a velocidade é negativa e a aceleração positiva se o movimento é acelerado.

    Achei a questão ambígua

  • achei que a aceleração seria negativa
  • carai confudi a e v

  • A aceleração é positiva pelo referencial! Como ele diz ''com uma velocidade de 30 m/s, no sentido contrário à orientação positiva da trajetória, e a sua aceleração é de 10 m/s2 no sentido positivo da trajetória.'' dá pra ver que a velocidade que vai ser negativa e a aceleração positiva! É como se um carro estivesse sendo puxado pra trás e tentando acelerar o carro pra frente!

  • O carro está com sua velocidade no sentido contrário ao do eixo positivo, entao V é negativo

    A aceleração está para o sentido do eixo positivo, então a é positivo.

    S = So + VoT + at²/2

    S = 40 - 40.4 + 10.16/2

    S = 40 - 120 + 80

    S = 0

  • O que confunde é "como o móvel pode estar em movimento acelerado se A e V tem sinais contrários"

  • entao nao faz a prova,amigo.Fica com a AFA ;)


ID
1909156
Banca
COMVEST - UNICAMP
Órgão
UNICAMP
Ano
2015
Provas
Disciplina
Física
Assuntos

A demanda por trens de alta velocidade tem crescido em todo o mundo. Uma preocupação importante no projeto desses trens é o conforto dos passageiros durante a aceleração. Sendo assim, considere que, em uma viagem de trem de alta velocidade, a aceleração experimentada pelos passageiros foi limitada a amax = 0,09g, onde g=10 m/s2 é a aceleração da gravidade. Se o trem acelera a partir do repouso com aceleração constante igual a amax, a distância mínima percorrida pelo trem para atingir uma velocidade de 1080 km/h corresponde a

Alternativas
Comentários
  • De acordo com o enunciado, tem-se:
    a) aceleração máxima do trem = 0,09g = 0,09 x 10m/s² = 0,9 m/s²
    b) deslocamento do trem: v² = vo² + 2aΔS
    v = 0
    vo = 1080 km/h = 300m/s
    a = 0,9 m/s²
    ΔS = ?
    Assim,
    300² = 0 + 2.0,9.ΔS
    ΔS = 90000 / 1,8 = 50000 m = 50 km

    Resposta C)


                                   


  • Torricelli : 

    v²=vo² + 2 a deltaS (Vovó + 2 antas Deltadas) 

    v² = vo² + 2 a delta S 

    - Antes de começar, preste atenção nas unidades. Note que a unidade de velocidade está em Km/h e a gravidade em m/s². É mais fácil converter de km/h - m/s . Basta dividir por 3,6 

    1080 / 3,6 é a mesma coisa que 10800/36 (melhor trabalhar com número sem vírgula). 
    Esse resultado nos dará = 300m/s.  Ok temos todos elementos da fórmula

    V = 300m/s
    Delta S = O que queremos descobrir
    Vo = 0 (saiu do repouso)
    aceleração máxima = 0,09g - ou - 0,9m/s² (já trocando o g por dez). 

     v² = vo² + 2 a delta S 
    300²=2 x 0,9 x Delta S 
    90000 = 1,8 Delta S 
    Delta S = 90.000 / 1,8 (para tirar a vírgula multiplica em cima e em baixo por 10) 
    Delta s = 900.000/18 
    Delta s = 50.000 metros. 
    Como as alternativas estão em KM basta dividir por 1000. 
    Resposta : 50Km 'C'


ID
1918219
Banca
Marinha
Órgão
CEM
Ano
2014
Provas
Disciplina
Física
Assuntos

Um ponto material movimenta-se no espaço com vetor posição dado por r(t) = (t2 i + cos t j + esen t k)m, onde t é medido em segundos. A aceleração desse ponto material no instante t = 0s é :

Alternativas
Comentários
  • r(t) = t^2 i + cos t j + e^(sen t) k

    r'(t) = 2t i + sen t j + (-cos t)e^(sen t) k

    r''(t) = 2 i + (-cos t) j + [(cos t)^2 * e^(sen t)] k

    r''(t) = aceleração

    r''(0) = 2 i - cos 0 j + [(cos 0)^2 * e^(sen 0)] k

    r''(0) = 2 i - 1 j + [(1)^2 * e^(0)] k

    r''(0) = 2 i - 1 j + 1 k

    Portanto letra B


ID
1919788
Banca
Marinha
Órgão
Quadro Complementar
Ano
2014
Provas
Disciplina
Física
Assuntos

Considerando que uma partícula move-se num plano de acordo com as equações de movimento x(t) = t 2 +3t+2ey(t) =4t,para t≥0, calcule o módulo da velocidade inicial e o módulo da velocidade mínima, respectivamente, e assinale a opção correta.

Alternativas
Comentários
  • Função Velocidade é a derivada da função posição.

    Em x(t), vx(t)=2t+3. Em y(t), vy(t)=4.

    Quando t=0, vx=3 e vy=4.

    Resultante é √3^2+4^2=5m/s.

    Como a função velocidade em y é constante e a em x é um reta crescente, quanto maior o tempo, maior será a componente de x. Logo, a velmín é a inicial, 5m/s.

    Gabarito D.


ID
1922683
Banca
PUC - GO
Órgão
PUC-GO
Ano
2015
Provas
Disciplina
Física
Assuntos

TEXTO 1

                              Queimada

À fúria da rubra língua

do fogo

na queimada

envolve e lambe

o campinzal

estiolado em focos

enos

sinal.

É um correr desesperado

de animais silvestres

o que vai, ali, pelo mundo

incendiado e fundo,

talvez,


como o canto da araponga

nos vãos da brisa!


Tambores na tempestade


[...]

E os tambores

  e os tambores

    e os tambores

soando na tempestade,

ao efêmero de sua eterna idade.


[...]

          Onde?

Eu vos contemplo

     à inércia do que me leva

     ao movimento


de naufragar-me

     eternamente

na secura de suas águas

   mais à frente!


Ó tambores

   ruflai

sacudi suas dores!


Eu

que não me sei

não me venho

   por ser

busco apenas ser somenos

no viver,

nada mais que isso!

(VIEIRA, Delermando. Os tambores da tempestade. Goiânia: Poligráfica, 2010. p. 164, 544, 552.)

O fragmento do Texto 1 “à inércia do que me leva / ao movimento” faz uma menção figurada a movimento. Uma partícula se movimenta ao longo de uma linha reta, obedecendo à função horária S = 80 + 30t - 5t2 , com S dado em metros e t em segundos. Sobre esse fenômeno são feitas as seguintes afirmações:

I-No intervalo de 0 a 2 segundos, o movimento é retilíneo progressivo retardado.

II-No intervalo de 0 a 8 segundos, a distância percorrida e o módulo do deslocamento da partícula são iguais.

III-Após 3 segundos, a partícula descreve um movimento retilíneo retrógrado retardado.

IV-A velocidade da partícula no instante t = 10 segundos terá um módulo igual a 70 m/s.

Com base nas sentenças anteriores, marque a alternativa em que todos os itens estão corretos:

Alternativas

ID
1946956
Banca
Marinha
Órgão
Quadro Complementar
Ano
2010
Provas
Disciplina
Física
Assuntos

As coordenadas x e y de uma partícula que se move no plano em função do tempo t são dadas por x = 2t2 + 3t e y = (t 3 /3) - 8 . Sabendo-se que x e y estão em metros e t em segundos, assinale a opção que apresenta, aproximadamente, o módulo da velocidade (em m/ s) e da aceleração (em m/ s2) , em t = 3s, respectivamente.

Alternativas

ID
1966612
Banca
Aeronáutica
Órgão
EEAR
Ano
2011
Provas
Disciplina
Física
Assuntos

Durante o pouso de um pequeno avião, num trecho reto de uma pista molhada, o piloto aciona os freios visando parar a aeronave. Quando o avião chegou neste trecho da pista, com uma velocidade de 108 km/h, os freios foram acionados e ele percorreu uma distância de 225 m até parar completamente. Admitindo uma desaceleração constante, o tempo gasto pela aeronave, desde acionar os freios até parar completamente, foi de _____ s.

Alternativas
Comentários
  • V²=VO²+2.A.DELTA S                       

    30²+2.A.250

    900+500A

    A= 900/500

    A= APROXIMADAMENTE 2,M/S²

    V=30-2T

    2T=30

    T= 30/2

    T=15 SEGUNDOS

    ACELERAÇÃO NEGATIVA POR CONTA DA TRAJETÓRIA

  • Pq que deltaS ( ou a Variação do espaço) é 250 se na questão fala que é 225 a distância que o avião percorre até parar?

  • Precisamos achar o tempo, mas primeiro precisamos descobrir a aceleração:

     

    V² = V0² + 2a . /\S

    0² = 30² + 2.a.225

    0 = 900 + 450a

    450a = -900

    a = -2 m/s² (temos uma desaceleração, logo o valor será negativo.)

     

    Agora é só achar o tempo:

     

    V = V0 + a.t

    0 = 30 -2.t

    -2t = -30

    t = 15 segundos

  • Devemos primeiro determinar a aceleração que é dada pela formula a=V/T sabemos que V inicial é 108 km/h convertendo em metros/segundo temos 108/3,6= 30 metros/segundo logo a=-30/T. O problema deixa claro que S final é igual a 225 e S inicial é igual a 0 logo temos a função horária do espaço como: 225=So+Vo x T + (-30/t) x t*2 Com o (-30/t) x t*2 dividido em forma de fração por dois 225= 0+30T -15t

    225= 15T

    225/15=T

    T= 15 segundos

    GABARITO A


ID
1972360
Banca
Aeronáutica
Órgão
EEAR
Ano
2010
Provas
Disciplina
Física
Assuntos

Durante o pouso de um pequeno avião, num trecho reto de uma pista molhada, o piloto aciona os freios visando parar a aeronave. Quando o avião chegou neste trecho da pista, com uma velocidade de 108 km/h, os freios foram acionados e ele percorreu uma distância de 225 m até parar completamente. Admitindo uma desaceleração constante, o tempo gasto pela aeronave, desde acionar os freios até parar completamente, foi de _____ s.

Alternativas
Comentários
  • V = veloc. final = 0 m/s 

    deslocamento = delta S = 225 m 

    Equação de Torricelli (ñ depende do tempo, que nesse problema eu desconheço): 

    V² = V0² + 2.a.delta S 

    0² = 30² + 2.a.225 

    900 + 450a = 0 

    a = -2m/s² ----> desaceleração de 2m/s² (por isso o sinal negativo!) 

    V = V0 + a.t 

    0 = 30 - 2t 

    2t = 30 

    t = 15 s

  • Dica: Quando desconhecer o Tempo e a Aceleração/Desaceleração, primeiro usa torricelli para descobrir a aceleração!

    V² = Vo² + 2aS

    0 = 30.30 + 2.a.225

    -900 = 450a

    a = -2

    Agora, usa V = Vo + at para achar o tempo!

    0 = 30 + (-2).T

    2T = 30

    T = 15

    EsPCEx 2020 BORA

  • como que você achou 2m/s somando 900 com 450?
  • ele não somou, ele dividiu amigo!


ID
2006548
Banca
Aeronáutica
Órgão
EEAR
Ano
2012
Provas
Disciplina
Física
Assuntos

Considere a função x = 4t – t2 onde (x) é a posição, em metros, de um ponto material em movimento retilíneo que varia em função do tempo (t), em segundos. Dentre as alternativas, assinale aquela que estabelece o instante, em segundos, em que a posição do ponto material é x = 0 m.

Alternativas
Comentários
  • t^2 + 4t - 0 = 0

    Isto é uma equação de segundo grau, logo para achar a função de t usa-se soma e produto ou bhaskara .

    -4 + 0 = -4

    -4 . 0 = 0

    t = (-4,0)

    Depois substitui os valores de t na função

    X = t^2 + 4t

    X = (-4)^2 + 4.(-4)

    X = 16 - 16

    X = 0

    X = t^2 + 4t

    X = (0)^2 + 4.(0)

    X = (0)^2 + 4.(0)

    X = 0 + 0

    X = 0

    Por uma questão lógica o tempo neste caso não poderá ser 0.

    Como não se tem tempo negativo multiplica-se o (-4) por -1 ficando desta forma:

    t = 4

  • Para 4.t-t^2=0

    4t=t^2

    Elimina o termo comum:

    t=4

    4.4=4^2

    16=16

    X=4.4-4^2

    X=16-16

    X=0

  • precisa nem de conta só usar a lógica:

    4 x t - t²

    o t² tem que ser um valor que resulta em 4 x t

    ou seja, 4 x 4 = 16 - 4² = 16 -16 = 0

    Gostou? acesse www.organizeconcursos.com.br

  • Fiz assim:

    x=4t-t²

    4t-t²=0

    Colocando em ordem fica:

    -t²+4t=0

    (para não complicar a nossa vida, multipliquei por (-1) para o primeiro termo não ficar negativo:

    -t²+4t=0 (-1)

    t²-4t=0

    Aqui eu fiz por soma e produto para achar as raízes:

    S= -B/A= -(-4)/1=4

    P=C/A= 0/1=0

    (não temos o valor de c então ele vai ser 0)

    As raízes só pedem ser (0, 4) porque multiplicado é 0.4=0 e que somados é 4+0=4

    Só fazer a substituição:

    (por 0)

    x=4t-t²

    x= 4(0)-0²

    x=0

    (tempo não pode ser 0)

    (por 4)

    x=4t-t²

    x=4.4- (4)²

    x= 16-16

    x=0

    Ou seja, o tempo terá que ser 4.

    Gabarito letra D

    Mas se você não quiser gastar esse tempo todo, pode ir substituindo os T pelos números da alternativas!

  • Brena Oliveira, só fazendo uma pequena correção em sua colocação. O tempo só não pode ser negativo, mas com certeza ele pode ser igua a zero.

  • t² - 4t = 0 coloca em evidência t(t-4)=0 raízes t =0 t = 4
  • Jesus te ama!

  • Gente, vai de Bhaskara !

    X = 4t - t²

    0 = 4t - t²

    a= 1 b= 4 c= 0

    ∆= 4² - 4.1.0

    ∆= 16

    X= -4 ± √16

    2.1

    X1= -4 + 4 = 0

    2

    X2= -4 -4 = -8 = -4

    2 2

    A primeira raiz encontrada não satisfaz a equação, e a segunda é negativa, então o que resta a se fazer é multiplicar o (-4) por (-1), que vai deixar positivo.

    -4. (-1) = 4

    X = 4


ID
2074819
Banca
Marinha
Órgão
EFOMM
Ano
2016
Provas
Disciplina
Física
Assuntos

Um trem deve partir de uma estação A e parar na estação B, distante 4 km de A. A aceleração e a desaceleração podem ser, no máximo, de 5,0 m/s2 , e a maior velocidade que o trem atinge é de 72 km/h. O tempo mínimo para o trem completar o percurso de A a B é, em minutos, de:

Alternativas
Comentários
  • De acordo com o enunciado, tem-se:
    4km = 4000m
    72km/h = 20m/s
    Assim,
    V = Vo + a.t
     20m/s = 0 + 5m/s² . t 
    t1 = 4s

    V² = Vo² + 2a.S
    400 = 0 + 10.S
    S = 40m

    D = 4000 - 2S
    D = 4000 - 80
    D = 3920m

    D = V . t
    3920 = 20 . t
    t2 = 196 s

    Finalizando,
    Tmín = 2 . t1 + t2
    Tmín = 2.4 + 196 = 204s = 3,4min

    Resposta E)

  • Primeiro vamos ajustar as unidades deixando em metros

    4km= 4000m  72km/h = 20m/s

    Para começarmos vamos calcular quanto tempo e quantos metros o trem anda até chegar a sua aceleração máxima 

    Para isso usaremos Vf = Vo + at

    20 = 0 + 5t

    t = 4

    Agora vamos descobrir a distância que ele percorre até chegar a sua velocidade máxima 

    D = Vo.t + at²

                     2

    D = 0.t + 5.16

                    2

    D = 40m

    Para desacelerar, ele também gastará o mesmo tempo e andará a mesma distância 

    Portanto, somando o período de aceleração e desaceleração, o trem ja andou 80m e levou 8 segundos

     

    Agora vamos calcular o tempo que ele leva para percorrer os 3920m restantes com uma velocidade máxima de 20m/s 

    Vm= DeltaS

             DeltaT

     

    20 = 3920

              t

    t = 196

    Agora é só somarmos o tempo de aceleração e desaceleração ( 8 segundos ) com o tempo que ele leva com a velcidade constante ( 196 segundos )

    Totalizando 204 segundos

    Como a questão pede o resultado em minutos, é só fazermos uma simples regra de 3

    1 min ----------- 60s

    x min ------------ 204



    X = 3,4 min  Alternativa e) 

  • Vm=DeltaS

    DeltaT

    20=4000(m)

    DeltaT

    DeltaT= 200s / 60 s (de cada minuto) = 3,3, ou seja, como pediu o mínimo, o valor próximo e 3,4 da resposta.

  • nathan a velocidade final vale 0

  • Complementando...

    Também podemos fazer por análise gráfica:

    A fim de realizar os 4000m em tempo mínimo, o trem deverá gastar o mínimo tempo possível para acelerar de 0 a 20m/s, usando a máxima aceleração, no ínicio e no final.

    Imagine um gráfico vxt, usaremos a máxima aceleração de 5m/s a cada 1s, o trem atinge a velocidade de 20m/s em 4s (t=v/a) => MRUV; e continua com essa velocidade durante mais x segundos => MRU; logo após sofre uma frenagem e a velocidade é reduzida de 20m/s para 0 m/s em mais 4s, quando finalmente o trem para.

    Observe que vc terá um gráfico com 3 áreas: A1 um triangulo de altura 20 e base 4, A2 um retângulo de altura 20 e base X e A3 = A1.

    Daí é só fazer A1+A2+A3 = (20*4)/2 + 20x + (20*4)/2 = 4000 => x= 196s

    Lembre que o gráfico de área = (NUMERICAMENTE) ao deslocamento (4000 m), por isso fiz isso.

    Assim, o tempo total gasto pelo trem para fazer esse percurso vale:

    4 + x + 4 = 4+196+4 = 204 s ou 3,4min


ID
2118262
Banca
MS CONCURSOS
Órgão
CBM-SC
Ano
2010
Provas
Disciplina
Física
Assuntos

“ A tendência de todas as coisas na natureza é o equilíbrio. O equilíbrio pode ocorrer na forma de REPOUSO ou de MOVIMENTO RETILÍNEO UNIFORME (MRU). Assim, se um corpo estiver em equilíbrio ele permanecerá em equilíbrio até que um agente externo o retire de seu estado de REPOUSO ou MRU, pois um corpo por si não pode desequilibrar-se.” Esta assertiva refere-­se a:

Alternativas
Comentários
  • Discordo, pois o fato de ser funcionário publico, é que deu a ele a oportunidade de entrar na repartição ao lado..

  • GABARITO A)

    1ª Lei de Newton ( princípio da inércia)... Um corpo tende a manter um equilíbrio ( estático ou dinâmico) a menos que uma força passe a agir sobre ele.

    2ª Lei de Newton ( princípio fundamental da dinâmica). A força resultante das forças que atuam em um corpo é igual à massa * aceleração... Ou seja, a força resultante é proporcional à aceleração, possuindo mesma direção e sentido.

    3ª Lei de Newton ( Princípio da ação e reação). Para toda ação há uma reação de mesma intensidade, mesma direção, porém de sentidos opostos. Lembrando que isso somente ocorre em corpos distintos.

  • Perfeito, Philerson.


ID
2121799
Banca
FUNIVERSA
Órgão
CBM-AP
Ano
2012
Provas
Disciplina
Física
Assuntos

Em um parque de diversões, um carro simples de montanha-russa desce pela primeira parte de um trilho a um vale com profundidade h1. Em seguida, sobe uma ladeira até o topo de uma montanha com altura h2 acima do fundo do vale. Considerando g a aceleração da gravidade, assinale a alternativa que indica a expressão da velocidade inicial (v0) mínima necessária para assegurar a chegada do carrinho ao topo da montanha

Alternativas
Comentários
  • fazendo por torricelli

    V²= Vo²+2a(S-So)

    0=Vo²+2g(H2-H1)

    Vo=Raiz²(2g(H2-H1))

  • Conservação da energia mecânica...

    EmecA= EmecB

    m * g * h1 + m * Vo²/2 = m * g *h2

    Vo²= 2g* ( h2 - h1)

    Vo= raiz de 2g* ( h2 - h1)


ID
2128126
Banca
FCC
Órgão
Copergás - PE
Ano
2016
Provas
Disciplina
Física
Assuntos

Um veículo parte do repouso com aceleração a = K1 − K2v2, sendo K1 a aceleração constante resultante da tração nas rodas e −K2v2 a aceleração devido ao arrasto aerodinâmico. A distância percorrida pelo veículo até atingir uma certa velocidade V é:

Alternativas
Comentários
  • Sabe aquela questão que no cespe tu corre? Pois é... kkkk


  • O exercício pede a distância que é a integral dupla da aceleração, ou seja, derivada de segunda ordem de uma das alternativas é igual a equação da aceleração.

    Derivada ln(x) = 1/x. Daí já descarta as alternativas a, d, e.

    A derivada segunda é em função de V.

    (V^2)' = 2V. Precisa de um 2 no denominador para cancelar. Portanto, Alternativa C.


ID
2148475
Banca
NC-UFPR
Órgão
UFPR
Ano
2015
Provas
Disciplina
Física
Assuntos

João e Alceu saíram de carro de Curitiba, para visitar o colega Mathias na cidade de Garuva. Partiram do km 90 e deslocaram-se até o km 120, quando João percebeu que esquecera os documentos do carro na casa de sua mãe. Mudam o sentido do movimento e retornam até o km 70. O deslocamento escalar e a distância efetivamente percorrida de carro por João e Alceu, em metros (m), são, respectivamente, de:

Alternativas
Comentários
  • ∆s = s - s0           km 70 - km 90= -20km    transformando para metro -20000m  (deslocamento escalar)

     

    km 90 ate km 120= 30km

    km 120 ate km 70=50 km                  30+50=80km  transformando para metro 80000m  (distancia)


ID
2166712
Banca
IBFC
Órgão
PM-MG
Ano
2015
Provas
Disciplina
Física
Assuntos

Considere que uma bala de 10 gramas de massa é disparada por um revólver a 300 m/s. Considere que uma bala exatamente igual é deixada cair do alto de prédio de 250 metros de altura. Assinale a alternativa que corresponde à relação entre a energia cinética imediatamente após a bala ser disparada (Ed) e no instante que a outra bala atinge o solo após a queda (Eq). Considerar g = 10 m/s2 e desprezar o atrito com o ar em ambos os casos.

Alternativas
Comentários
  • Ed = (m . v² )/ 2

    Ed = (10 . 300²) / 2

    Ed = 450.000

    Eq = m . g. h

    Eq = 10 . 10 . 250

    Eq = 25.000

     

    Ed / Eq = 450.000 / 25.000 = 18

  • Energia Cinética / Energia potencial gravitacional

    Ec= m.v²/2

    Ec= 10x300²/2

    Ec= 450.000 J

    Epg = m. g. h

    Epg = 10x10x250

    Epg = 25.000 J

    Relação = 450.000/25.000= 18

    LETRA C

    APMBB


ID
2178361
Banca
COPS-UEL
Órgão
CBM-PR
Ano
2010
Provas
Disciplina
Física
Assuntos

Quando os freios de um automóvel são acionados bruscamente, ele desliza com as rodas travadas numa desaceleração constante. Num teste de reflexo de frenagem, um motorista dirigiu o automóvel sem ingerir bebida alcoólica (situação 1), e em outro teste dirigiu levemente alcoolizado (situação 2). Em ambas as situações o automóvel era mantido numa velocidade constante de 90 km/h e, num mesmo ponto de referência, o motorista deveria acionar os freios bruscamente.
Considerando que o reflexo do motorista levemente alcoolizado é retardado em 1 segundo, quantos metros o automóvel da situação 2 ficará distante do automóvel da situação 1 ?

Alternativas
Comentários
  • O primeiro (e mais importante) passo é passar a velocidade de km/h para m/s:

    90/3,6 = 25 m/s

    Agora acabou o exercício!

    A distância estre as situações analisadas será justamente a distância que o carro percorre nos 1 segundo que o motorista alcoolizado demorara para acionar os freios.

    ΔS = v.t

    ΔS = 25.1

    ΔS = 25 metros

    GABARITO: LETRA C

  • 25 m/s

    25 metros por segundo, muito boa a análise do Victor

  • 1° passo transformar de km/h para m/s

    90km/h -> 25m/s

    2° passo d=v.t

    situação 1

    D=20T

    situação 2

    D=20.(T+1)

    D=20T+25

    ou seja, situação 2 percorre 25 metros a mais do que na situação 1


ID
2208448
Banca
PUC - RJ
Órgão
PUC - RJ
Ano
2016
Provas
Disciplina
Física
Assuntos

Um carro saiu da posição xi = 0 km e percorreu uma estrada retilínea e horizontal até xf = 10 km. Entre 0 km e 5 km, sua velocidade foi 60 km/h e, entre 5 km e 10 km, sua velocidade foi 30 km/h.

Calcule, em km/h, a velocidade média para percorrer os 10 km totais.


Alternativas
Comentários
  • Entre 0km e 5 km, o tempo gasto foi: T = 5/60 = 1/12 horas

    Entre 5 km e 10km, o tempo gasto foi: T = 5/30 = 1/6 horas

    Tempo total: 1/12 + 1/6 =1/4 horas

    Velocidade média = distância percorrida total/tempo total = 10/1/4= 40km/h

  • Vamos lá...

     

    1) Tempo gasto do kilômetro 0 ao kilômetro 5:

    Vm = Δs/Δt

    60 = 5/Δt

    Δt = 5/60

    Δt = 1/12

     

    2)Tempo gasto do kilômetro 5 ao kilômetro 10:

    Vm = Δs/Δt

    30 = 5/Δt

    Δt = 5/30

    Δt = 1/6

     

    Soma dos tempos:

    1/12 + 1/6 = 

    1/4 h

     

    Encontrando a velocidade média:

    Vm = Δs/Δt

    Vm = 10 / 1/4 

    Vm = 10 . 4/1

    Vm = 40km/h

     

    Gabarito: C

  • Δs/Δt

    Δs1= v.t

    5= 60.t

    t= 1/12 x 60 = 5 minutos

    Δs2= é justamente a metade, logo, como foi mais lento, será 10 minutos

    » o tempo total foi 15minutos, mas o enunciado pede em horas 15/60= 0,25h

    VM= ΔsTotal/ΔtTotal

    VM= 10/0,25

    VM=40Km/h

    LETRA C

    APMBB


ID
2231041
Banca
BIO-RIO
Órgão
ETAM
Ano
2011
Provas
Disciplina
Física
Assuntos

Cinco segundos após sua partida, um barco alcança a velocidade de 10 km/h. A aceleração média foi de:

Alternativas
Comentários
  • Aceleração = Velocidade/Tempo

    10km/h ÷3,6= 2,77m/s

    2,77m/s÷ 5segundos

    0,55m/s²

    Letra D


ID
2236186
Banca
CETRO
Órgão
AEB
Ano
2014
Provas
Disciplina
Física
Assuntos

Uma partícula move ao longo de um eixo s com umaaceleração variada no tempo dada pela função α(t) = 3t2 +4t+6. Sabendo que a velocidade inicial da partícula é 20 e sua posição inicial 10 e usando integração, é correto afirmar que a função de posição da partícula é

Alternativas
Comentários
  • Sabemos que derivando a função do espaço em relção ao tempo, obtemos a velocidade, isto é, (s(t))'=v(t).

    Ao derivar a velocidade em relação ao tempo, temos a aceleração, isto é, (v(t))'=a(t).

    Ou seja se integrarmos a aceleração, temos a velocidade. Integrando agora a velocidade, temos o espaço, isto é, integral da aceleração é t^3+2t^2+6t+20 (20 é a contante, pois foi dada no exericio), integrando a velocidade temos (t^4)/4+(2t^3)/3+3t^2+20t+10 (10 é a contante, pois foi dada no exericio). logo alternativa E).

    Note que se usarmos s(t)=s0+v0t+at^2/2 deveria dar certo.

     

  • Pessoal estou com bastante dúvida , pfvr alguém responda essa questão. 

  • Faz duas integrações para achar a fórmula da posição. 

    Depois acrescenta os dados que foram dados. Velocidade Inicial e Posição Inicial.

    Lembrando da fórmula é: S = So + VT

  • Considerando que é para entrar na AEB, questão muito fácil. Basta fazer duas integrações em t.

    como são polinômios, usa-se somente a integral de u^n du = [u^(n+1)]/n+1. Lembrando também que a integral da soma é a soma das integrais, isso é suficiente para resolver a questão.


    Quem não entender como se resolve a questão, não fique triste! É uma questão para quem fez curso na área de exatas e aprendeu cálculo diferencial e integral e também física I.


ID
2245930
Banca
FUNRIO
Órgão
CBM-GO
Ano
2016
Provas
Disciplina
Física
Assuntos

Em reportagem sobre acidentes de trânsito no Brasil, publicada em 18/03/2015, o sociólogo e especialista em segurança no trânsito Eduardo Biavati alertou sobre o perigo do uso do celular ao volante. Segundo Biavati: Antes o uso do celular se restringia a ligações de voz, que demandava basicamente uma das mãos. A situação de risco foi maximizada com as mudanças do próprio uso do smartphone. Para você digitar qualquer letra, além da mão, é preciso olhar para a tela. Ou seja, a distração é mais profunda: perde-se o contato visual e o mecânico. Um estudo do NHTSA,departamento de Trânsito dos Estados Unidos, revela que o uso de dispositivos móveis ao volante aumenta em até 400% o risco de acidente. Um risco muito maior do que o causado pela embriaguez! 

Suponha que dois carros trafeguem a 90km/h na mesma pista, separados por uma distância de seguimento de 50m. De repente, em virtude de retenções na via, o carro da frente freia constantemente a uma taxa de 5,0m/s2 até parar. Porém, o motorista do carro de trás está distraído, lendo uma mensagem no Whatsapp e demora 2,2s para perceber a luz do freio do carro da frente e mais 0,30s para reagir e efetivamente pisar fundo no freio, provocando uma forte desaceleração constante de 7,5m/s2

Considerando essas informações, pode-se afirmar que uma colisão 

Alternativas
Comentários
  • Dividindo a questão em etapas temos:

    I) Carro da frente: Vo = 90km/h = 25 m/s       a = -5m/s^2

    V = Vo + at      ==>     0 = 25 - 5t      ==>      t = 5s (tempo de frenagem)

    S = So + Vot + at^2/2    ==>   S = 0 + 25*5 + (-5)*5^2/2     ==>      S = 62,5m (distância percorrida pelo carro durante a frenagem)

    ----------------------------------------------------------------------------------------------------

    II) Carro de trás: Vo = 25 m/s         a = -7,5 m/s^2

    V = Vo + at      ==>     0 = 25 - 7,5t      ==>      t = 3,3s (tempo de frenagem)

    S = So + Vot + at^2/2    ==>   S = 0 + 25*3,3 + (-7,5)*3,3^2/2     ==>      S = 41,7m (distância percorrida pelo carro durante a frenagem)

    Distância percorrida durante a distração: d = Vot      ==>  d = 25*2,5      ==>     d = 62,5m     (tempo de distração = 2,2 + 0,3)

    Distância total percorrida pelo carro de trás: S = 41,7 + 62,5     ==>     S = 104,2

    -----------------------------------------------------------------------------------------------------

    Para ocorrer a batida a distância percorrida pelo carro de trás teria de ser maior do que a distância percorrida pelo carro da frente + a distância entre ambos que era de 50m, totalizando 112,5m. Não foi o que ocorreu, portanto gabarito é letra "E".

  • bom comentário Felipe, só não entendi como tu chegou nesse tempo de frenagem do carro de trás: 

    0 = 25 - 7,5t      ==>      t = 3,3s (tempo de frenagem)

  • Vendo o pessoal calcular o tempo ... Não é necessário . Apenas por Torricelli para os dois carros é possível chegar no resultado. A única variável relevante para decidir sobre a colisão é a distância

  • Dados do enunciado:

    Ambos carros espaçados por 50 metros.

    Carro de trás:

    Vo = 90 km/h = 25 m/s

    T de reação = 2.5 s

    a = -7.5 m/s²

    Carro da Frente:

    Vo = 90 km/h = 25 m/s

    a = -7.5 m/s²

    --------------------------------------------------------------

    Percebam que o tempo é sim uma variável importante nessa questão pelo fato do veículo de trás começar a frenar 2,5s depois do veículo da frente.

    --------------------------------------------------------------

    Sendo So = 50 m para o veículo da frente --> Agora vamos identificar o So do veículo de trás:

    So = 25 * 2.5 = 62.5 m (a partir daqui que o veículo de trás começa a frenar)

    --------------------------------------------------------------

    Vamos identificar quanto tempo o veículo da frente leva para parar completamente

    t = 25 / 5 = 5 segundos

    --------------------------------------------------------------

    Vamos aplicar agora a função horária do espaço para os dois veículos e comparar aonde cada qual parou:

    S = So + Vot + at²/2

    Veículo de trás

    S = 62.5 + 25(2.5) + (-7.5)*(2.5)² / 2 = 102.5 m

    Veículo da frente

    S = 50 + 25(5) + (-5)*(5)² / 2 = 112.5 m

    --------------------------------------------------------------

    Conclusão:

    Alternativa E --> Pois, ainda sobrarão 10m de distância entre um e outro.


ID
2250370
Banca
FCC
Órgão
Copergás - PE
Ano
2016
Provas
Disciplina
Física
Assuntos

Um veículo parte do repouso com aceleração a = K1 − K2v2, sendo K1 a aceleração constante resultante da tração nas rodas e −K2v2 a aceleração devido ao arrasto aerodinâmico. A distância percorrida pelo veículo até atingir uma certa velocidade V é:

Alternativas
Comentários
  • Alguem explica?

  • Nessa hora o povo da PRF chora. kkkkk

  • Acredito que tenha que integrar duas vezes a equação dada. Mas não lembro mais como faz isso. Haha

  • Maldade isso. Tem q manipular a equação pra achar dx/dv, separar e integrar. Engenheiro so resolve isso quando estuda eq. diferenciais na faculdade, depois nunca mais.


ID
2297761
Banca
SELECON
Órgão
ETAM
Ano
2017
Provas
Disciplina
Física
Assuntos

Observa-se o movimento de um carro de corrida, logo após a largada. Verifica-se que durante 5 s ele percorre 150 m e adquire uma velocidade de 45 m/s. Supondo que seu movimento tenha sido uniformemente acelerado, no instante em que se iniciaram as observações a velocidade do carro era:

Alternativas
Comentários
  • fazendo um gráfico, seu formato equivale a de um trapézio

    sendo eixo x o tempo em segundos e o eixo y a variação da velocidade.

    No eixo y, temos um intervalo que inicia o Vo, pelos dados do enunciado a sua velocidade inicial não será zero.

    E mais acima tem-se a velocidade após os 5 segundos, que é de 45 m/s

     

    (V + Vo).t/2 = S
    45 + Vo = 2.150/5

    Vo = 15 m/s

    letra B

  • Não estou conseguindo entender a solução feita pelo Flávio Gomes, alguém poderia refazer a questão? Obrigado !!

     

  • Anna Barros, fazendo o desenho da trajetória equivale a área do trapézio. Como Flávio Gomes mostra na resposta dele, logo,

    Área = (B+b).h/2, 

    B=45m/s

    b = Vo, que é diferente de zero

    h = 5s

    Área = 150 m 

     

  • CONSEGUI FAZER DE UM JEITO MAIS FÁCIL QUE VI NUMA DAS AULAS DE FÍSICA DO ESTRATÉGIA CONCURSOS PROFESSOR VÍNICIUS SILVA

    Vm = VF(velocidade final) + VI(velocidade inicial)

                ____________________________________    

                                               2

    LEMBRANDO QUE Vm = VARIAÇÃO DE ESPAÇO / VARIAÇÃO DE TEMPO 

    FICA ASSIM :

     

    VARIAÇÃO DE ESPAÇO / VARIAÇÃO DE TEMPO =   VF  +  VI

                                                                                               _____

                                                                                                  2

     

    RESOLVENDO A QUESTÃO: 

    VARIAÇÃO DE ESPAÇO NA QUESTÃO = 150 m

    VARIAÇÃO DE TEMPO NA QUESTÃO = 5 s

    VELOCIDADE NA QUESTÃO =44 m / s

     

    Aplicando a fórmula:

     

    150               45 + VI( QUE AINDA NÃO SABEMOS)

    ___        =   ________________________________

    5                                        2(DA FORMULA)

     

    MEIO PELOS EXTREMOS

     

    300 = 225 + 5VI

    5VI= 75

    VI =75/5

    VI = 15 m/s

     

  •     S total / tempo total = Veloci final - Veloci inicial 

          150 / 5  = 45 -V inicial

             30 = 45 - V inicial

         V inicial = 45 - 30

         V inicial = 15 m/s  

  • Utilizei    v=vo + at     e      s = so + vot + at2/2

    pelos dados da questão e fazendo-se as devidas substituições, teremos: 

    vo + 5a = 45 

    5vo + 25a/2 = 150

    resolvendo o sistema acima, encontra-se: 

    a=6m/s2

    vo=15 m/s

  • Basta fazer utilizando-se do gráfico V x T.

    Será formado um trapézio.

    Área do trapézio é: A=(B+b).h/2

    150=(45+x).5/2

    300=225+5x

    300-225=5x

    75=5x

    x=75/5

    x=15 m/s

  • A velocidade média de um M.R.U.V pode ser dada pela média das velocidades:

    Vm= Vo+Vf/2

    S/t= Vo+Vf/2

    150/5=Vo+45/2

    Vo= 60 -45

    Vo = 15 m/s


ID
2304958
Banca
PUC - GO
Órgão
PUC-GO
Ano
2016
Provas
Disciplina
Física
Assuntos

TEXTO 3

                          Escalada para o inferno

Iniciava-se ali, meu estágio no inferno. A ardida solidão corroía cada passo que eu dava. Via crucis vivida aos seis anos de idade, ao sol das duas horas. Vermelhidão por todos os lados daquela rua íngreme e poeirenta. Meus olhos pediam socorro mas só encontravam uma infinitude de terra e desolação. Tentava acompanhar os passos de meu pai. E eles eram enormes. Não só os passos mas as pernas. Meus olhos olhavam duplamente: para os passos e para as pernas e não alcançavam nem um nem outro. Apenas se defrontavam com um vazio empoeirado que entrava no meu ser inteiro. Eu queria chorar mas tinha medo. Tropeçava a cada tentativa de correr para alcançar meu pai. E eu tinha medo de ter medo. E eu tinha medo de chorar. E era um sofrimento com todos os vórtices de agonia. À minha frente, até onde meus olhos conseguiram enxergar, estavam os pés e as pernas de meu pai que iam firmes subindo subindo subindo sem cessar. À minha volta eu podia ver e sentir a terra vermelha e minha vida envolta num turbilhão de desespero. Na verdade eu não sabia muito bem para onde estava indo. Eu era bestializado nos meus próprios passos. Nas minhas próprias pernas. Tinha a impressão que o ponto de chegada era aquele redemoinho em que me encontrava e que dele nunca mais sairia. Na ânsia de ir sem querer ir eu gaguejava no caminhar. E olhava com sofreguidão para os meus pés e via ainda com mais aflição que os bicos de meus sapatos novos estavam sujos daquela poeira impregnante, vasculhante, suja. Eu sempre gostei de sapatos. Eu sempre gostei de sapatos novos. Novos e luzidios. E eles estavam sujos. Cobertos de poeira. E a subida prosseguia inalterada. Tentava olhar para o alto e só conseguia ver os enormes joelhos de meu pai que dobravam num ritmo compassado. Via suas pernas e seus pés. E só. Sentia, lá no fundo, um desejo calado de dizer alguma coisa. De dizer-lhe que parasse. Que fosse mais devagar. Que me amparasse. Mas esse desejo era um calo na minha pequenina garganta que jamais seria curado. E eu prossegui ao extremo de meus limites. Tinha de acontecer: desamarrou o cadarço de meu sapato. A loucura do sol das duas horas parece ter se engraçado pelo meu desatino. Tudo ficou muito mais quente. Tudo ficou mais empoeirado e muito mais vermelho. O desatino me levou ao choro. Não sei se chorei ou se choraminguei. Só sei que dei índices de que eu precisava de meu pai. E ele atendeu. Voltou-se para mim e viu que estava pisando no cadarço. Que estava prestes a cair. Então me socorreu. Olhou-me nos olhos com a expressão casmurra. Levou suas enormes mãos aos meus pés e amarrou o cadarço firmemente com um intrincado nó. A cena me levou a um estado de cegueira anestésica tão intensa que sofri uma espécie de amnésia passageira. Estado de torpor. Quando dei por mim, já tinha chegado ao meu destino: cadeira do barbeiro. Alta, prepotente e giratória. Ele, o barbeiro, cabeça enorme, mãos enormes, enormes unhas, sorriso nos lábios dos quais surgiam grandes caninos. Ele portava enorme máquina que apontava em minha direção. E ouvi a voz do pai: pode tirar quase tudo! deixa só um pouco em cima! Ali, finalmente, para lembrar Rimbaud, ia se encerrar meu estágio no inferno.

(GONÇALVES, Aguinaldo. Das estampas. São Paulo: Nankin, 2013. p. 45-46.)

Para chegar à barbearia, o menino de 6 anos, personagem do Texto 3, percorreu um longo caminho sobre ruas de chão batido. Supondo-se que a distância a percorrer pelo garoto, no intervalo [0, t], forme uma progressão aritmética de termo geral at , com t dado em minutos; supondo-se ainda que a soma das distâncias a percorrer nos instantes de tempo de dois, cinco e seis minutos dê 400 metros, e que a metade desse valor é obtida pela soma das distâncias a serem percorridas nos instantes de três, seis e sete minutos, pode-se afirmar que a distância percorrida pelo menino até a barbearia é de aproximadamente (assinale a resposta correta):

Alternativas
Comentários
  • Alguém pode me ajudar nessa?


ID
2335255
Banca
FUNCAB
Órgão
POLITEC-MT
Ano
2013
Provas
Disciplina
Física
Assuntos

Que gráfico abaixo melhor representa a aceleração em função do tempo no caso de um movimento uniformemente acelerado?

Alternativas
Comentários
  • No MRUV a aceleração é constante. O que varia é a velocidade. Desse modo, a letra E representa o gráfico.

  • No Movimento Uniformemente Variado, a aceleração é constante e diferente de zero, logo, a função da velocidade é uma função constate, e o gráfico que representa essa função é uma reta paralela ao eixo dos tempos.

     

    Gabarito LETRA D


ID
2381482
Banca
IBFC
Órgão
POLÍCIA CIENTÍFICA-PR
Ano
2017
Provas
Disciplina
Física
Assuntos

A posição de um veículo em uma pista sem curvas varia com o tempo, segundo a função:

S = 11 + 2t (s em metros e t em segundos).

Assinale a alternativa que apresenta, respectivamente, a posição inicial e a velocidade.

Alternativas
Comentários
  • Função da Posição em Relação ao Tempo

     

    S = So + V x t

     

    Só olhar a fórmula e ver os correspondentes. Está muito claro que 11 m correspondem à posição inicial (So) e 2 m/s à velocidade (V).

     

    Gabarito A

  • A função da posição no decorrer do tempos, se dá através da equação abaixo:

    S = S0 + V . T

    S = Espaço Final

    S0 = Espaco Inicial

    V = velocidade

    T =Tempo

    Ao  analizarmos a equação acima e a descrita no exercicio ( S = 11 + 2t ), podemos identificar que:

    S0 = 11 m/s

    V = 2 m/s

     

    Gabarito Letra A!

     

  • S= So + V. T    < SORVETE >

    So= 11m 

    V= 2 m/s

    So > Espaço Inicial 

    V > Velocidade  


ID
2381491
Banca
IBFC
Órgão
POLÍCIA CIENTÍFICA-PR
Ano
2017
Provas
Disciplina
Física
Assuntos

Um veículo parte do repouso e atinge a velocidade de 100 m/s em 10s. Então, sua aceleração média nesse intervalo de tempo será de:

Alternativas
Comentários
  • Fórmula da Aceleração Média

     

    a = Δv/Δt

     

    Onde: a = aceleração (?)

             Δv = variação da velocidade (100 m/s)

             Δt = variação do tempo (10 s)

     

    a = 100/10

    a = 10 m/s²

     

    Gabarito E

  • Fórmula da Aceleração Média

    A = Δv/Δt

    A = Aceleração Média

    Δv = variação da velocidade

    Δt = variação do tempo

    Um veículo parte do repouso e atinge a velocidade de 100 m/s em 10s. Então, sua aceleração média nesse intervalo de tempo será de: 

    A = Δv/Δt

    A = (100 - 0) / (10 - 0)

    A = 100 / 10

    A = 10 m/s²

     

    Gabarito Letra E!

  • Macete:

    Vou vomitar aquela tapioca

    V = Vo +at

     

    Basta substituir os valores com os dados do enunciado:

    100 = 0 + 10a

    a = 100/10 = 10 m/s²

     

    GABARITO: E

  • Pelo menos essa acertei.Era questão de honra. PQP

  • vovô mais atraente...kkkk


ID
2381494
Banca
IBFC
Órgão
POLÍCIA CIENTÍFICA-PR
Ano
2017
Provas
Disciplina
Física
Assuntos

Um ponto material em movimento adquire velocidade que obedece à função V = 20 – 4 t (no SI). Assinale a alternativa que indica, respectivamente, a velocidade inicial e a aceleração.

Alternativas
Comentários
  • Função da Velocidade em Relação ao Tempo

     

    V = Vo + a x t

     

    Percebe-se com clareza que 20 m/s é a velocidade inicial e -4 m/s² é a aceleração.

     

    Gabarito C

  • tem nada de claro ai. Seria bastante interessante um maior empenho por parte da staff do qc para resolver as questões com explicações por professores dessas matérias. Ta errado e ai ?

    Ninguém tem esclarecimento

     

  • A função da velocidade no decorrer do tempos, se dá através da equação abaixo:

    V = Vo + A . T

    V = Velocidade Final

    V0 = Velocidade Inicial

    A = Aceleração

    T =Tempo

    Ao  analizarmos a equação acima e a descrita no exercicio ( V = 20 – 4 t), podemos identificar que:

    V0 = 20 m/s

    A = -4 m/s

     

    Gabarito Letra A!

  • Como já disse o Sr. Shelking: use a fórmula da Velocidade x Tempo.

    V = Vo + a.t    (velocidade = velocidade inicial + aceleração * tempo)
    V = 20 – 4 t    (observe que na função dada na questão o "Vo" foi subtituído por 20 e o "a" foi substituído por -4)

    ou seja, Vo = 20 e a = -4

  • V = V0 + a x t; V = 20 - 4t

    V0 = 20m/s

    a = -4m/s

    GABARITO: C


ID
2381497
Banca
IBFC
Órgão
POLÍCIA CIENTÍFICA-PR
Ano
2017
Provas
Disciplina
Física
Assuntos

Um ponto material em movimento adquire velocidade que obedece à função V = 60 – 20 t (no SI). Desse modo, a velocidade no instante 2s seria igual a:

Alternativas
Comentários
  • Novamente se trata da função da velocidade em função do tempo. Basta colocar no t o valor de 2s e resolver:

     

    V = 60 - 20 x 2

    V = 60 - 40

    V = 20 m/s

     

    Gabarito C

  • A função da velocidade no decorrer do tempos, se dá através da equação abaixo:

    V = Vo + A . T

    V = Velocidade Final

    V0 = Velocidade Inicial

    A = Aceleração

    T =Tempo

    Ao  analizarmos a equação acima e a descrita no exercicio ( V = 60 – 20 t), Desse modo, a velocidade no instante 2s:

    V = Vo + A . T

    V = 60 – 20 t

    T = 2s

    V = 60 – 20 . 2

    V = 60 – 40

    V = 20 m/s²

     

    Gabarito Letra C!

  • só substituir o T pelo 2 e fazer a conta!

  • Muito engraçado as questões de física quase não tem gente que comenta ou responde essas questões ...
    Quando vamos para direito penal tem 320 comentários, isso mostra que as pessoas não dão muito foco nas exatas e na prova isso pode ser um super diferencial, estou me aventurando por aqui kkkk "Parece a Deep Web" 

  • fácil demaissss juniuuuuu

     

     

    Quanto ao comentário abaixo \/ @Claudio Alves 

    É bem isso... tem que saber de tudo um pouco.. quem quiser que fique só no direito!!

    abcs

  • Neste caso é substituir na fórmula para achar o resultado...

    V = 60 – 20 t > Instante 2s...ou seja, onde está o "t" colocamos "2", assim:

    V = 60 - 20.2

    V = 60 - 40

    V = 20m/s

    Gab Letra C


ID
2381500
Banca
IBFC
Órgão
POLÍCIA CIENTÍFICA-PR
Ano
2017
Provas
Disciplina
Física
Assuntos

Um automóvel está parado diante de um semáforo. Imediatamente após o semáforo abrir, um ônibus o ultrapassa com velocidade constante de 40 m/s. Neste mesmo instante, o automóvel arranca com uma aceleração de 8 m/s2 . Determine em quanto tempo o automóvel alcançará o ônibus e marque a alternativa correta.

Alternativas
Comentários
  • É preciso igualar o espaço em que eles se encontram.

     

    Ônibus = Carro

    MU = MUV

     

    So + vt = So + Vo x t + at²/2

    o + 40t = 0 + 0xt + 8 x t²/2

    40t = 4t² (eliminamos uma incógnita [t] de cada lado da igualdade)

    40 = 4t

    40/4 = t

    10 s = t

     

    Obs.: é possível resolver a equação do segundo grau 4t² - 40t = 0. Obter-se-á o mesmo valor, 10.

     

    Gabarito B

  • Fiz  de outra maneira ...

    CARRO --->

    V(inicial) = 0

    a = 8m/s² 

    Utilizando a Função da Velocidade -- > 40 = 0 +8T -->  T= 5 segundos (esse é o tempo que o carro, com uma aceleração de 8m/s² levaria para, no mínimo ,igualar a velocidade do ônibus = 40m/s)

    Porém , nesses 5 segundos o onibus não ficou parado ele percorreu certa distância então...

    Função do Espaço ---> S = 0 +40.5 --> S = 200m 

    RESPOSTA :

    O carro precisa percorrer 200m para alcançar o ônibus  então...

    200 = 0 +40T --> T= 5s

    Como o carro precisou de 5s para igualar a velocidade do onibus precisará de mais 5s para percorrer a distância que o onibus percorreu durante esses segundos gastos para igualar a velocidade , ou seja T= 10s

     

                                                                                        

  • Fazer a equação do Sorvetão para ambos, igualando as expressões resultantes. Depois é só resolver a equação de 2º grau.

  • 1 (Ônibus) = MRU
    2 (Carro) = MRUV

     

    Iguala a equação do MRU do Ônibus com o MRUV do Carro.

     

    S1 = So + V.t       (onde So = 0)
    S2 = So + Vo.t + (a . t^2) / 2 (onde So = 0, e Vo.t = 0)

     

    Teremos: S1=S2 --->  V.t = (a . t^2) / 2    ----> 40 x t = (8 x t^2)/2

     

    Resolvendo temos que:   t = 10 s.

     

     

  • Obrigado ao Sr. Shelking e a todos que deixam seus cometários aqui. Física para mim é o Ó, mas chegaremos juntos...

  • Vou colocar da forma que eu aprendi, a resolução em sí a galera já fez.


    Quando tiver uma questão falando de um carro ultrapassar o outro parado, e quanto tempo levaria do carro parado alcançar o que passou, deverá usar 2 fórmulas, um para cada veículo, veja:

    Veículo que estava parado é MUV, pois teve uma aceleração, usa-se: S= So + Vo.t + a.t² / 2.

    Veículo que ultrapassou está em movimento constante, aí será MU, usa-se o sorvete = S = So + V.T


    Depois é só igualar os dois resultados e encontrar o tempo.

  • quem sabe resolver por derivadas?


ID
2381503
Banca
IBFC
Órgão
POLÍCIA CIENTÍFICA-PR
Ano
2017
Provas
Disciplina
Física
Assuntos

Um carro trafega a uma velocidade de 36 km/h. Quando freado, para somente após percorrer 25 metros. Nessas condições, a aceleração introduzida pelos freios será de:

Alternativas
Comentários
  • Como o carro é freado, tenha em mente desde já que a aceleração será negativa e que sua velocidade final será zero, já que ele freia e finalmente para. Sabendo disso, convertamos km/h para m/s, dividindo o valor de 36 km/h por 3,6. Obteremos 10 m/s. Agora, usemos a Equação de Torricelli para encontrarmos a resposta.

     

    V² = V²o + 2aΔS

     

    Onde: V = velocidade final (0)

              Vo = velocidade inicial (10 m/s)

              a = aceleração

              ΔS = variaçaõ do espaço (25 m)

     

    0 = 10² + 2 x a x 25

    -100 = 50a

    -100/50 = a

    -2 m/s² = a

     

    Gabarito D

     

  • Equação de Torricelli

    V² = V0² + 2 . A . Δs

    V = Velocidade Final = 0 m/s

    V0 = Velocidade Inicial = 36 km/h / 3,6 = 10 m/s

    A = Aceleração 

    Δs = variação do espaço = 25 m

     

    V² = V0² + 2 . A . Δs

    0² = 10² + 2 . A . 25

    0² = 100 + 50. A

    A = - 100 / 50

    A = - 2 m/s²

     

    Gabarito Letra D!

  • Quando a questão não menciona o TEMPO utilizamos a equação de Torricelli.

     

    Vf² = Vo² + 2aD

    Vf² = 0 (velocidade final)

    Vo = 36 => 36/3,6 = 10m/s (velocidade inicial)

    a = aceleração

    D = Distância percorrida

     

    Vf² = Vo² + 2aD

    0² = 10² + 2.a.25

    0² = 100 + 50.a

    - 100/50 = a

    -2 = a

    a = -2m/s²

     


ID
2381506
Banca
IBFC
Órgão
POLÍCIA CIENTÍFICA-PR
Ano
2017
Provas
Disciplina
Física
Assuntos

Um avião durante a decolagem percorre uma distância de 1800 metros a partir do repouso, com uma aceleração constante de 25 m/s2 . A velocidade do avião durante a decolagem foi de:

Alternativas
Comentários
  • Equação de Torriceli (M.R.U.V)

     

    V² = V²o + 2aΔs

     

    Onde: V = velocidade final (?)

              Vo = velocidade inicial (0)

              a = aceleração (25 m/s²)

              Δs = variação do espaço (1800 m)

     

    V² = 0 + 2 x 25 x 1800

    V² = 50 x 1800

    V² = 90.000

    V = √90.000

    V = 300 m/s 

     

    Gabarito B

      

  • Equação de Torricelli

    V² = V0² + 2 . A . Δs

    V = Velocidade Final

    V0 = Velocidade Inicial = 0 m/s

    A = Aceleração = 25 m/s²

    Δs = variação do espaço = 1800 m

     

    V² = V0² + 2 . A . Δs

    V² = 0² + 2 . 25 . 1800

    V² = 90000

    V = 300 m/s

     

    Gabarito Letra B!

     

  • O que o físico faz quando não tem tempo???

    Equação de Torriceli =]

  • Para ajudar nos calculos fiz assim:

     

    V²= 25.3600

    √ V²= √25.√3600

    não sei se é possivel essa operação mas o resultafoi igual


ID
2389708
Banca
IBFC
Órgão
POLÍCIA CIENTÍFICA-PR
Ano
2017
Provas
Disciplina
Física
Assuntos

Um trem percorre um trajeto de modo que sua velocidade aumenta ao longo do tempo. Em determinado momento, sua velocidade escalar é de 90 km/h e, 5,0 segundos depois, sua velocidade escalar média é de 126 km/h. A aceleração escalar média do trem entre os dois instantes é igual a:

Alternativas
Comentários
  • 126km/h  / 3,6= 35m/s

    90km/h  /3,6= 25m/s

     

    V= Vo+a.T

    35=25+a.5

    10=a5

    10/5=2m/s²

  • Outra forma de resolver:

     

    a = dV/dT

    a = ((126km/h-90km/h)/3,6)/5s

    a = 10m/s / 5s

    a = 2m/s²

  • Gabarito: B

    Comentário:


    Transformando os valores:


    90 km/h --► 25 m/s

    126 --► 35 m/s


    Aceleração = Velocidade/Tempo

    Velocidade = 35 - 25 = 10 m/s

    Tempo = 5 - 0 = 5 s


    Aceleração = 10/5 = 2 m/s²

  • Primeiro tem que converter de km/h para m/s

    Dados:

    V = 35 m/s (126 km/h ÷ 3,6)

    Vo = 25 m/s (90 km/h ÷ 3,6)

    a = ?

    t = 5,0 seg.

    Eq. Aceleração:

    V = Vo + a . t

    35 = 25 + a . 5

    35 - 25 = a . 5

    10 = 5a

    a = 10/5

    a = 2 m/s² (Gab. "B")

  • V1=90km/h

    V2=126km/h

    126-90=36km/h

    Passar para m/s:

    36÷3,6=10

    10÷5=2m/S^2

  • V1=90km/h

    V2=126km/h

    126-90=36km/h

    Passar para m/s:

    36÷3,6=10

    10÷5=2m/S^2

  • Igualar no S.I.

    90 / 3,6 = 25 m/s

    126 / 3,6 = 35 m/s

    Delta de v = 35 - 25 = 10

    Fórmula da aceleração: v / t

    Substituir e achar a aceleração

    v / t = 10 / 5 = 2 m/s²

  • V = 35 m/s (126 km/h ÷ 3,6)

    Vo = 25 m/s (90 km/h ÷ 3,6)

    T= 5,0 seg.

    Am= ?

    1° passo= achar Am1 ( Aceleração Média )

    Am= 25÷5= 5

    2° passo= Achar Am2

    Am=35÷5= 7

    Depois é só subtrair Am1 e Am2 pra achar a Aceleração Média.

    7-5= 2m/s²


ID
2389711
Banca
IBFC
Órgão
POLÍCIA CIENTÍFICA-PR
Ano
2017
Provas
Disciplina
Física
Assuntos

Um trem percorre um trajeto com velocidade escalar constante de 90 km/h. Em um determinado instante, o sistema de freio do trem é acionado, provocando uma desaceleração constante de 5,0 m/s² até o trem parar. A distância percorrida durante a frenagem foi de:

Alternativas
Comentários
  • gabarito E.

     

    90km/h= 25m/s

     

    V²=Vo²+2.a(D.intervalo)

    0²=625+2.-5.D

    10d=625

    D=625/10= 62,5m

  • Vo --> velocidade inicial = 90 km/h = 25m/s

    V ---> velocidade final ---> o trem freou até parar, é = 0

    a ---> aceleração = 5 m/s²

    Δs ---> distância percorrida durante a frenagem = ?

     

    Não falou em tempo ---> Equação de Torricelli

    V² = Vo² + 2.a.Δs

    25² = 0 + 2.5.Δs

    625 = 10Δs

    Δs = 62,5m

  •                                                                     V² = Vo² + 2 • a • ∆S                       90/3,6 = 25

                                                                        0  = 25² + 2 • 5 • ∆S

                                                                        0  = 625 + 2 • 5 • ∆S

                                                                        0  = 625 + 10∆S

                                                                  625  = 10a

                                                                         a = 625 / 10          a = 62,5m/s.

  • Dados:

    V = 0 (o trem para totalmente)

    Vo = 25m/s (tem que dividir por 3,6 para passar pra m/s 90÷3,6 = 25m/s)

    a = 5,0m/s²

    d = ?

    Eq. Torricelli (sempre que não mencionar o tempo)

     ̶V̶²̶ = Vo² + 2 . a . d

    0 = (25)² + 2 . (5,0) . d

    d = 625 + 10d

    d= 625 / 10

    d= 62,5 m (gab."E")

  • Vo = 90km/h= 25m/s

    Vf = 0

    a = - 5 m/s² (desaceleração)

    Equação de Torricelli

    V² = Vo² + 2.a.Δs

    0 = 25² + 2 . (-5) . Δs

    0 = 625 -10 . Δs

    10 . Δs = 625

    Δs =62,5

  • A velocidade inicial sempre tem que estar em m/s???

  • ΔS = Vo²/ 2.a


ID
2389717
Banca
IBFC
Órgão
POLÍCIA CIENTÍFICA-PR
Ano
2017
Provas
Disciplina
Física
Assuntos

Um veículo em movimento adquire velocidade representada pela função V = 30 - 2t (no SI). Nessas condições, a velocidade do veículo decorridos 2,5 segundos, é de:

Alternativas
Comentários
  • aplicação direta da fórmula: V= Vo + a*t

    V= 30 -2*2,5

    V= 25 m/s

  • perceba que a função dada tem a mesma estrutura da fórmula da Função Horária da Velocidade:

    V = Vo + a . t 

    V = 30 - 2 . t

    Vo = 30 m/s

    a = -2 m/s²

    t = 2,5 s

     

    V = 30 + (-2) . 2,5 = 30 + (-5) = 25 m/s

     

                                                                                      "Brasil acima de tudo, Deus acima de todos."

  • V = 30- 2.t

    t = 2,5 s

     

    Então ; 

    V = 30-2.2,5  > V= 30-5 > V=25 m/s 

  • Gabarito: B

    Comentário: basta aplicar direto o valor na equação.


    V = 30 - 2t


    Para t=2,5, temos:

    V = 30 - 2.2,5

    V = 30 - 5

    V = 25 m/s


  • Substitui t por 2,5. Sem mistério.

    v = 30 - 2 . 2,5

    v = 30 - 5

    v = 25


ID
2390161
Banca
COPESE - UFJF
Órgão
UFJF
Ano
2017
Provas
Disciplina
Física
Assuntos

Um corpo efetua um movimento retilíneo obedecendo à função horária:

S(t) = 5 (m/s2 ) t2 + 4 (m), onde t é dado em segundos.

Considerando que esse corpo possui uma massa de 5 Kg qual é o valor da força aplicada ao movimento desse corpo?

Alternativas
Comentários
  • s= so + vot + at^2 /2

    a acerelacao é igual a a 10 ( 5*2) que é dada pela função horária.

    F=m*a = 5*10= 50 resposta letra E

  • Na verdade não usei fórmula para resolver essa questão. Apenas parti do pressuposto de que 1 Kg corresponde a 9,8 N (o qual podemos arredondar para 10). Nessa óptica, um corpo de massa com valor 5 Kg, cuja questão quer saber em N, é: 5.10 = 50 N. Logo, a assertiva correta é a que consta na letra E (50,0 N).

    "Desiste não, "carai". O melhor está por vir". - Mano Brown.

  • A fórmula de deslocamento é S= So+Vot+(at²)/2, por isso o valor da aceleração para essa questão está dividido por 2, e para encontramos a aceleração para usar na fórmula de força, multiplicamos o 5 dado na formula por 2, assim teremos:

    F=m.a

    F=5.10

    F=50N

  • Fernando Dias, você deu sorte que nesse exercício a aceleração é a mesma da gravidade, pois seu raciocínio está errado. A força é massa x aceleração, só pode considerar 10, como você fez, caso essa força seja a gravidade (P=m*g).

  • v =dS/dt = 10t

    a=dv/dt = 10

    F = m.a = 50N

  • DERIVA A VELOCIDADE EM RELAÇÃO AO TEMPO E OBTEMOS A ACELERAÇÃO. MULTIPLICA ESSA ACELERAÇÃO OBTIDA PELA MASSA E OBTEMOS A FORÇA

    50N


ID
2414143
Banca
Quadrix
Órgão
SEDF
Ano
2017
Provas
Disciplina
Física
Assuntos

Segundo o princípio da conservação da energia, a energia mecânica total de um sistema que não sofre a ação de forças externas permanece constante. Assim, a energia é conservada quando a energia mecânica total é inalterada. Com base no princípio da conservação da energia, julgue o item a seguir, considerando que a aceleração da gravidade (g) seja igual a 10 m / s2 .

Uma bola de 380 g foi arremessada verticalmente, de baixo para cima, com velocidade inicial de módulo igual a 10 m/s. A altura máxima (h), em metros, que a bola atinge, supondo que a resistência do ar seja desprezível, está situada no intervalo 4,8m<h< 5,1 m.

Alternativas
Comentários
  • GABARITO CERTO

     

    A energia cinética imposta ao corpo no momento inicial é convertida inteiramente em energia potencial quando o objeto alcança o ponto máximo da trajetória.

     

    mv²/2 = mgh

    h = v²/2g

    h = 100/20 = 5m

  • Basta utilizarmos a equação de Torricelli,

    como a questão fala que a resistência do ar é desprezível, não nos interessará a massa

     

    V² = Vo² + 2aH

    Obs: a velocidade final é 0 pois é quando ele atinge a altura máxima

    Obs 2 : a aceleração será negativa porque como o corpo foi jogado para cima ele está desacelerando

     

    0² = 10² + 2 x (-10) x H

    0 = 100 - 20H

    20H = 100

    H = 5 metros

     

    GABARITO: CERTO

  • Correto ! 

    Fiz com a seguinte fórmula: Hmáx = Vo² / 2 . g 

    Hmáx = 100 / 20 = 5 

     

  • Dados:

    vo (vel. inic.) = 10m/s

    v (vel. fin.) = 0 (pois ao atingir a altura máx. a velocidade será zero)

    g (gravid.) = 10 m/s²

    d (dist.) = ?

    Fórmula (Torricelli)

    v² = vo² + 2. g. d

     ̶(̶0̶)̶²̶ ̶= (10)² + 2 . (10) . d

    100 = 20.d

    d = 100/20

    d = 5 m

    Gab. CORRETO (esta no intervalo mencionado 4,8m<h< 5,1 m.)

  • Pode tbm achar a energia cinética: m.v²/2

    vai ter 19 de ec.

    daí aplica na fórmula de energia gravitacional, que tem altura.

    epg= mgh (eu peguei minha gata hoje) Prof. Juliano Broggio

    como energia se conserva, então a cinética e gravitacional são as mesmas.

    19= 0.38 . 10. h

    h= 5

    obs: 380 g = 0,38 kg (sistema internacional).

  • mv²/2 = m.g.h

    v² = 2.g.h

    100=2.10.h

    h = 5m

    --------------------------------- ou

    vf²=vi² + 2.a.D

    10² = 0 + 2.10.D

    D = 5m


ID
2422216
Banca
IFB
Órgão
IFB
Ano
2017
Provas
Disciplina
Física
Assuntos

Uma partícula se move na direção horizontal de modo que a sua posição é dada pela seguinte função: X(t) = 4.t²+2.t³/3, onde t está em segundos e X em metros. Podemos afirmar que a aceleração da partícula no instante t = 0.5s é igual a:

Alternativas
Comentários
  • Derivada

    Formula:  x(t) = 4.t² + (2.t³)/3

    X'(t) = 2 x 4t + (3 x 2t²) / 3

    X'(t) = 8t + (6t²) / 3

    X'(t) = 8 + 2t²

    X'(t) = 8 + 2

    X'(t) = 10m/s²

  • NAO CONSEGUI RESOLVER.

    1º analisando a formula dada pelo exercício, tratei como se tratasse da formula de deslocamento D=Vot+at²/2

    2º usei a formula dada pelo enunciado x(t) = 4.t² + (2.t³)/3 e substituí o t por 0,5 s, já que ele queria nesse tempo >> x= 0,416m

    3º pensei em usar a formula da aceleração a=delta D/deltaT >> a= 0,832 m/s²

    =(

     

  • Derivando a 1ª vez, acha a variaçao de velocidade, derivando a 2 vez, acha se a variaçao da aceleraçao

    X(T)= 4t² + 2T³/3

    X(T)=8T + (6T²/3)

    X(T)= 8+4T

    x(T)= 8+ 4(.5)

    X(T)=10M

     

  • Bruno,

    Você não pode utilizar a analogia com S = So + Vot +at²/2 , pois a trajetória da partícula é dada por um expressão CÚBICA, e não QUADRÁTICA.

    Por exemplo, se a expressão fosse 4t² + 2t/3 , vc poderia fazer a comparação!

     

    Sendo assim, a única maneira de descobrir a aceleração dessa partícula em certo instante é fazendo a segunda derivada da posição, como a Marcela Fragoso fez.......pois se temos a equação da posição de uma partícula, X(t),  a primeira derivada nos dá V(t) , velocidade, e a segunda derivada nos dá a(t), aceleração:

    X(t) = 4t² + 2t³/3  

    dx/dt  = V(t) = 8t + 2t²

    d²x/dt² = a(t) = 8 + 4 . t

     

    Substituindo t = 0,5 s , temos a = 10 m/s²

  • Regra de derivação 

    para calcula a velocidade instantânea é a DERIVADA PRIMEIRA DA POSIÇÃO EM FUNÇÃO DO TEMPO

    para calcula a acelração é a DERIVAÇÃO PRIMEIRA DA VELOCIDADE EM FUNÇÃO DO TEMPO OU A SEGUNDA DA POSIÇÃO 

    ACELARAÇÃO É A DERIVAÇÃO DA VELOCIDADE 

    VELOCIDADE É A DERIVAÇÃO DA POSIÇÃO.

  • Gabarito: B

    Comentário:

    Basta derivar duas vezes a equação para encontrar a equação da aceleração.



    X(t) = 4.t² + (2.t³)/3


    1ª derivada: Velocidade


    X'(t) = 2 x 4t + (3 x 2t²) / 3

    X'(t) = 8t + (6t²) / 3

    X'(t) = 8t + 2t²


    2ª derivada: Aceleração

    X''(t) = (8t + 2t²)'

    X''(t) = 8 + 4t


    Agora substituindo 0,5 no lugar de t, temos:


    X''(0,5) = 8 + 4.0,5

    X''(0,5) = 8 + 2

    X''(0,5) = 10 m/s²


ID
2422234
Banca
IFB
Órgão
IFB
Ano
2017
Provas
Disciplina
Física
Assuntos

Uma esfera maciça de densidade d1 e massa m se encontra dentro de um recipiente que contém um líquido de densidade d2 , de modo que d2 =4.d1 . A esfera está presa no fundo deste recipiente por um fio. A distância da esfera até a superfície do líquido é H. Considere que o diâmetro da esfera é muito menor que H. Se o fio for cortado, desconsiderando as forças de viscosidade entre o líquido e a esfera e que aceleração da gravidade é g, podemos afirmar que o tempo gasto para que a esfera chegue até a superfície do líquido é dado pela expressão:

Alternativas
Comentários
  • Esfera está subindo,pois a densidade do fluido é maior do que a da esfera. Logo, usando a segunda lei de newton, tem-se.

    Empuxo - peso = m_esfx(aceleração)

    aceleração=3g       (1)

    Usando a seguinte equação:

    H =H_o +v_ot +1/2xaceleração.t^2     (2)

    Substituindo (1) em (2), obtém-se .

    t=raiz quadrada(2H/3g)

     

     

  • Por favor daria para descrever melhor? porque aceleraçao da 3g?nao consegui entender! a resoluçao esta bem sucinta demais!!

  • Fr = ma

    E-P=ma

    m2g-mg=ma

    d2Vg-mg=ma

    4d1Vg-mg=ma

    4mg-mg=ma

    3mg=ma -> a =3g

  • De acordo com a 2ª lei de Newton, Fr = m.a. Assim, temos no sistema com a corda partida que o empuxo leva a esfera pra cima e o peso é pra baixo tornando o sistema:

    Empuxo - Peso = m.a => d4.Vesf.g - d1.Vesf.g = d1.Vesf.g.a

    Onde:

    d4 = densidade do líquido

    Vesf = volume da esfera

    g = aceleração da gravidade

    d1 = densidade da esfera

    a = aceleração resultante

    Cancelando os termos possíveis, temos que a = 3 m/s², e não 3g.

    Obs.: o volume utilizado no empuxo é o volume de fluido deslocado. Como a esfera está totalmente submersa, o volume de fluido que foi deslocado foi o próprio volume da esfera.


ID
2427892
Banca
PUC-PR
Órgão
PUC - PR
Ano
2015
Provas
Disciplina
Física
Assuntos

Um automóvel parte do repouso em uma via plana, onde desenvolve movimento retilíneo uniformemente variado. Ao se deslocar 4,0 m a partir do ponto de repouso, ele passa por uma placa sinalizadora de trânsito e, 4,0 s depois, passa por outra placa sinalizadora 12 m adiante. Qual a aceleração desenvolvida pelo automóvel?

Alternativas
Comentários
  • O que temos?

    V = ?
    Vo = 0
    S = 12 m
    So = 4 m
    t = 4 s

     

    S = So + vt
    12 = 4 + v. 4
    8 = 4v
    v = 8/4
    v = 2 m/s

     

    Como já sabemos velocidade do veículo, iremos encontrar a aceleração.

    V = Vo + at
    2 = 0 + a.4
    a = 2/4
    a = 0,5 m/s²

     

    Gabarito: A

  • Tiago Gil. A questão afirma que é MRUV, mas você utilizou a fórmula de MRU e depois MRUV para calcular o segundo trecho.

    O resultado final está certo, mas seu cálculo não.


    Solução da questão: https://www.youtube.com/watch?v=QtCwz08m3Vk

  • Thiago Gil, pelo que eu entendi do seu cálculo, você considerou a velocidade do automóvel quando ele cruza a primeira placa como 0...mas como é 0 se ele está em movimento (ele passou pela primeira placa em movimento) ??

  • por que nao podemos usar a formula S=Szero+Vzero.T+a.T²/2?

  • Primeiro vamos descobrir a velocidade quando o carro alcança a primeira placa:

    Vf^2 = V^2 + 2.A.ΔU

    Vf^2 = 0 + 2.A.4

    Vf^2 = 8A

    Vf = raiz(8A)

    Descobrimos a velocidade do carro quando está na primeira placa.

    Agora vamos aplicar na formula Sf = S + v.t + a.t^2/2. Os dados que temos do carro quando está na primeira placa e quando está na segunda placa:

    Sf = S + Vf.t + a.t^2/2.

    16 = 4 + raiz(8A).4 + A.4^2/2

    12 = 4.raiz(8A) + 8A

    Divide os 2 membros por 4.

    3 = raiz(8A) + 2A

    3 - 2A = raiz(8A)

    Eleva os 2 membros ao quadrado.

    8A = 9 - 12A + 4A^2

    4A^2 -20A + 9 = 0

    Aplica Bashkara

    Descobrimos as raízes A= 0,5 e A''= 4,5

    Das alternativas a única que aparece é a A = 0,5m/s^2


ID
2445850
Banca
Aeronáutica
Órgão
EEAR
Ano
2017
Provas
Disciplina
Física
Assuntos

Em um porta-aviões as aeronaves pousam em uma pista útil de 100 m. Se a velocidade com que o avião toca a pista de tal embarcação é de aproximadamente 252 Km/h, determine o módulo da sua desaceleração média, em m/s:

Alternativas
Comentários
  • transformando km para m      252/3,6   = 70m/s

     

    V²=Vo² +2a. D

    0= 70² + 2.a. 100

    4900= 200a

    4900/200= 24,5

  • Se o problema quer a desaceleração, na questão deveria estar a unidade de medida sendo m/s2 e não somente m/s que é velocidade.

     

    Ao meu ver, questão deve ser anulada.

  • Convertendo 252km/h para m/s ---> 252/3,6 = 70m/s

    Aplica Torricelli

    V² = Vo² +2aΔS

    0² = 70² + 2.a.100

    -200a = 4900

    a = -24,5 m/s² (como está em módulo, não considera o sinal)

    Gab: B

  • Otávio Augusto, é somente um erro de digitação.

  • como se o m/s2 fosse motivo de errar o calculo para anular.

  • Como não temos tempo usaremos Torricelli

    V^2=V(inicial)^2 +2.a.∆s

    0=70^2 + 2.a.100 obs:v=0 porque está desacelerando!

    -200a=4900

    a=4900/-200

    a= -24,5m/s^2

    Como ele pede o módulo consideramos o sinal positivo somente.


ID
2445907
Banca
Aeronáutica
Órgão
EEAR
Ano
2017
Provas
Disciplina
Física
Assuntos

Um garoto chuta uma bola de futebol de 400g exercendo sobre ela uma força de 20N. Determine quanto tempo, em segundos, essa força deve atuar sobre a bola para que ela saia do repouso e atinja uma velocidade de 10 m/s.

Alternativas
Comentários
  • Pode ser resolvida usando a 2ª Lei de Newton que diz que a força resultante é igual a massa vezes a aceleração. (FR = m.a).

    Temos o valor da força que é 20 N, a massa da bola que é de 400g, ou 0,4 kg, porém não temos o valor expresso da aceleração; mas sabemos que a aceleração é a variação da velocidade sobre o tempo e sabemos que a variação da velocidade, nesse caso vale 10, já que a velocidade inicial da bola era zero e após a aplicação da força, tornou-se 10 m/s², só nos falta o tempo em que essa variação de velocidade aconteceu, que é o que a questão nos pede.

    Colocando na fórmula temos:

    FR= m.a ----->  20 = 0,4 . 10/t ----->  20.t = 0,4.10 -----> 20.t = 4 -----> t = 4/20, logo t = 0,2.

  • Podemos usar o teorema do impulso-------------------> Impulso=variação da quantidade de movimento------------->I=∆Q

    Não Esquueçam de transformas as unidades

    I=m.Vf - m.V.i-------> I=0,4.10 - 0,4.0-----------------> I=4N/s

    I=F.∆T

    4=20.∆T--------->T=0.2s

  • F - 20N                                                      F = m . a -----------------(a = v/t)

    m - 400g = (0,4 KG)                                 20 = 0,4 . 10/t

    v - 10 m/s                                                 t = 4/20

    t - ?                                                           t = 0,2 s

  • 1- Ache a aceleração por meio da fórmula FR=M.A

    2- Depois joga os dados na fórmula V=VO+AT

  • v=10 m/s

    m=400g --> 0,4kg

    F=20N

    t=?

    -----------------------------------

    FR= m * a

    20= 0,4 * a

    a= 20/0,4

    a= 50m/s^2

    -----------------------------------

    v=vo + a*t

    10= 0 + 50t

    t= 10/50

    t=0,2s

  • tá safe

  • fiz pelo teorema do impulso! GAB:b

  • Da pra fazer pelo impulso

    Quantidade de movimento = Impulso

    ( Q = M x V ) = ( I = F x /\T)

    M x V = F x /\T

    0,4 x 10 = 20 x /\T

    4/20 = /\T

    /\T = 0,2

  • F = m.a

    a = v/t

    F = m.(v/t)


ID
2555281
Banca
INEP
Órgão
ENEM
Ano
2017
Provas
Disciplina
Física
Assuntos

Um motorista que atende a uma chamada de celular é levado à desatenção, aumentando a possibilidade de acidentes ocorrerem em razão do aumento de seu tempo de reação. Considere dois motoristas, o primeiro atento e o segundo utilizando o celular enquanto dirige. Eles aceleram seus carros inicialmente a 1,00 m/s2. Em resposta a uma emergência, freiam com uma desaceleração igual a 5,00 m/s2.


O motorista atento aciona o freio à velocidade de 14,0 m/s, enquanto o desatento, em situação análoga, leva 1,00 segundo a mais para iniciar a frenagem.


Que distância o motorista desatento percorre a mais do que o motorista atento, até a parada total dos carros?

Alternativas
Comentários
  • Por aproximação e Raciocínio lógico matemático (Podemos já que as alternativas tem números bem distantes umas das outras,quase)
    5m/s2-1ms2= 4m/s2
    14/4=3,5
    15/4= 3,75 (guarda ai)

    3.75-3,5=0,25

    3,75-0,25= 3,5

    logo 14m( que ele andou em 1s e depois freiou) + 3,5 = 17,5

    E 17,5 éaproximadamente 17,4 (resposta da questão)

    Estudar lógica ajuda um pouco, se bem usada.

    Letra E
     

  • N sei se esta certo, mas pensei em movimento relativo. Desta forma o motorista atento é o referencial sendo sua posiçãoinicial e seu tempo iguais a 0. 

    Este seria o deslocamento a mais a partir do ponto em que o motorista atento parou.

    No motorista desatento a gente usa a equção horária da posição, ficando assim:

    S = 12 *  1 + 5 * 1^2 / 2 => 17,5

     

  • Vamos usar a equação de Torricelli porque não tem medida do tempo, por exemplo: minutos, horas, dias ou anos. É necessário ter um entendimento do texto. 

    (A) O motorista atento e motorista desatento aceleram seus carros inicialmente a 1,00 m/s2.

    Começando pelo atento: 14²= 0 + 2.1.ΔS1 

                                            196= 0 + 2.ΔS1

                                            196= 2.ΔS1

                                            ΔS1= 196/2= 98

    Agora vamos ver o motorista desatento: 15²= 0 + 2.1.ΔS2

                                                                     225= 0 + 2.ΔS2

                                                                     225= 2.ΔS2

                                                                     ΔS2= 225/2= 112,5.  

    (B)  O motorista atento e motorista desatento freiam com uma desaceleração igual a 5,00 m/s2. 

    Começando pelo atento: 14²= 0 - 2.5.ΔS1 (foi usado o sinal - porque é desaceleração. Se for aceleração, usa o sinal +. Tome cuidado! 

                                            196= 0 - 10.ΔS1

                                            196= 10.ΔS1

                                            ΔS1= 196/10= 19.6

    Agora vamos ver o motorista desatento: 15²= 0 - 2.5.ΔS1

                                                                     225 = 0 - 10. ΔS1

                                                                     225 = 10.ΔS1

                                                                     ΔS1= 225/10= 22.5. 

    Agora é só somar ΔS do motorista atento: 98+19.6= 117.6. Agora some ΔS do motorista desatento: 112.5 + 22.5= 135.  

    O texto está pedindo: Que distância o motorista desatento percorre a mais do que o motorista atento, até a parada total dos carros? 135 - 117.6= 17.4. Letra E. 

  • O motorista atento começa a desacelerar com 5,00 m/s² a partir de 14 m/s. Usando a equação de Torricelli,

    0 = 14²-2.5.ΔS1

    ΔS1 = 19,6 m.

    No caso do motorista desatento, pode-se separar o movimento em duas partes:

    i) enquanto ainda acelera a 1,00 m/s² a partir de 14 m/s chegando a 15 m/s, o deslocamento nesse trecho será

    15²= 14²-2.1.ΔS2’

    ΔS2’ = 14,5 m,

    ii) a partir daí, freia à 5,00 m/s², percorrendo um deslocamento

    0 = 15²-2.5.ΔS2’’

    ΔS2’’ = 22,5m.

    Logo, o desatento percorre no total 22,5 m + 14,5 m = 37 m, percorrendo a mais que o atento 37 m – 19,6 m = 17,4 m.

  • Vo Atento v= vo -at

    0=14-5t

    t=14/5

    então o espaço percorrido é s= s0 +vot -at²/2

    s= 0 + 14.14/5 -5.(14/5)²/2

    = 19.6

    Agora o desatento.

    Ele passa 1 segundo ainda acelerando antes de perceber.

    então o espaço será a velocidade média dividido pelo tempo.

    vm= (14+15)/2= 14,5 14,5/1segundo = 14,5

    Agora ele começa a ter o comportamento do outro desacelerando e repetimos o processo porém com a velocidade final 15m/s

    0=15-5t

    t=3s

    então o espaço percorrido é s= s0 +vot -at²/2

    s= 14,5 + 15.3 -5.3²/2

    = 14,5 +45 -45/2= 14,5 +45/2= 14,5 + 22,5= 37

    A variação é um menos o outro então ficamos com 37-19.6= 17,4

  • Motorista atento:

    0^2 = 14^2 - 2 x 5 x S

    S = 14^2/10

    S = 196/10

    S = 19,6 m

    Motorista desatento:

    S = So + Vot + at^2/2

    S = 0 + 14 x 1 x 1/2 => S = 14,5 m => V = Vo + at => V = 14 + 1 x 1 = 15 m/s

    V^2 = Vo^2 + 2 x a x s

    0^2 = 15^2 - 2 x 5 x S

    0^2 = 225 - 10S

    10S = 225

    S = 225/10

    S = 22,5 => 22,5 + 14,5 = 37 m

    37 m - 19,6 m = 17,4 m

    Letra E

  • Atento: a=v/t -> 1=14/t ->> t=14seg

    -5=-14/t ->> t= 2,8

    tempo total = 16,8

    Gráfico velocidade por tempo: área=d

    area do triângulo ( base(tempo)x altura ( velocidade)/2 —> D=16,8x14/2= 117,6

    Desatento:

    demorou um segundo a mais, ou seja velocidade 15m/s e tempo 15 seg

    Desacelerara: -5=-15/t —>> t= 3

    Tempo total: 18 seg

    D=18x15/2= 135

    D2-D1=17,4

  • Por gráfico fica bem mais rápido.

    Não consigo desenhar aqui, mas seria velocidade x tempo. Bastaria projetar a velocidade inicial de frenagem no eixo X 14m/s e 15m/s (velocidade) e o tempo que levou para a total parada no eixo Y 16,8s e 18s (tempo). Teremos dois triângulos e basta calcular a área de cada um e depois a subtração.

    O tempo total de parada é possível por meio V= v0+at para ambos.

  • Sendo bem sincero? Dava pra 'matar' essa questão sem cálculos se seu tempo estivesse apertado para solucionar.

    Se eles estavam a 14m/s e ele demorou 1 seg a mais para começar a frear, logo ele teria 14m + algo.

    Eliminava A,B,C.

    15m eu achei muito pouco, tempo de deslocamento de 1 segundo: 14m + tempo de frenagem = 17,4m faria mais sentido.

  • Resolvendo por Torricelli:

    Cara atento:

    V = 14 m/s

    Vo = 0

    a = 1

    ΔS = x

    (14)² = (0)² + 2*1*ΔS

    196/2 = ΔS

    ΔS = 98 m

    Cara desatento:

    V = 15 m/s

    Vo = 0

    a = 1

    ΔS = x

    (15)² = (0)² + 2*1*ΔS

    225/2 = ΔS

    ΔS = 112,5 m

    Agora, precisamos ver o ΔS da frenagem:

    Cara atento:

    V = 0 (0 pois ele irá parar, desaceleração)

    Vo = 14 m/s

    a = - 5 m/s² (sinal negativo, desascelerando)

    ΔS = x

    (0)² = (14)² + 2*(-5)*ΔS

    -196 = -10*ΔS

    ΔS = 19,6 m

    Cara desatento:

    V = 0

    Vo = 15 m/s

    a = - 5 m/s²

    ΔS = x

    (0)² = (15)² + 2*(-5)*ΔS

    -225 = - 10*ΔS

    ΔS = 22,5

    Somando os dois:

    Cara atento: 98 + 19,6 = 117,6 m

    Cara desatento: 112,5 + 22,5 = 135 m

    Diferença: 135 - 117,6 = 17,4 m

  • RESOLUÇÃO DETALHADA

    *MOTORISTA ATENTO:

    TEMPO PERCORRIDO DE 0 M/S PRA 14 M/S:

    -já que a aceleração é de 1 m/s² e a velocidade inicial é de 0 m/s, são 14 segundos pra chegar a 14 m/s.

    -sabendo disso, não precisa usar VOAT aqui.

    ESPAÇO PERCORRIDO DE 0 M/S PRA 14 M/S:

    S=So+Vo.t+a.t²/2

    S=0+0.14+1.14²/2= 98 m

    -a velocidade usada é a velocidade inicial, por isso é 0 m/s.

    -o tempo usado é o tempo final (até alcançar a velocidade final, 14 m/s).

    -a aceleração usada é a aceleração impressa pra ir de 0 m/s até 14 m/s, no caso, 1 m/s².

    TEMPO PRA IR DE 14 M/S PRA 0 M/S:

    -é preciso saber o tempo gasto pra ir de 14 m/s pra 0 m/s.

    -se usa VOAT já que temos todos os dados, menos o tempo:

    Vf=0, Vo=14, a=-5.

    -14 m/s é a velocidade inicial nesse caso, já que vai de 14 m/s pra 0 m/s (velocidade final).

    -a aceleração usada é a da freagem (-5 m/s), já que o motorista vai de 14 m/s pra 0 m/s.

    V=Vo+a.t

    0=14-5.t

    5t=14

    t=14/5= 2,8 s

    ESPAÇO PERCORRIDO PRA IR DE 14 M/S PRA 0 M/S:

    S=So+Vo.t+a.t²/2

    S=98+14.2,8-5.2,8²/2

    S=98+39,2-19,6

    S=117,6 m

    -se usa o tempo final necessário pra ir de 14 m/s pra 0 m/s, no caso, 2,8 s.

    -a posição inicial é o espaço percorrido de 0 m/s pra 14 m/s obtido ali em cima, no caso, 98 m.

    -essa posição inicial é onde o carro está no momento em que começa a frear.

    -caso alguém se pergunte, o resultado (117,6 m) é maior que a posição inicial (98 m) porque, ainda que haja a aceleração negativa/"desaceleração" de -5 m/s², o carro ainda continua tendo velocidade (percorrendo espaço) enquanto ela não chega a 0 m/s.

    •o valor de 39,2 é o que seria percorrido e somado ao espaço inicial (98 m) caso não houvesse desacelaração, o resultado seria:

    98+39,2=137,2

    •como há desaceleração, a velocidade não é constante, por isso o valor desse espaço (137,2 m) é diminuído, ficando 117,6 m.

    •esse valor considera o impacto da aceleração negativa sobre a velocidade positiva, por isso esse valor é obtido.

    *MOTORISTA DESATENTO:

    TEMPO PERCORRIDO DE 0 M/S PRA 15 M/S:

    -já que a aceleração é de 1 m/s² e 1 segundo a mais é percorrido em relação ao outro motorista, são 15 segundos pra chegar a 15 m/s.

    ESPAÇO PERCORRIDO DE 0 M/S PRA 15 M/S:

    S=So+Vo.t+a.t²/2

    S=0+0.15+1.15²/2= 112,5 m

    TEMPO PRA IR DE 15 M/S PRA 0 M/S:

    V=Vo+a.t

    0=15-5.t

    5t=15

    t=15/5= 3 s

    ESPAÇO PERCORRIDO PRA IR DE 15 M/S PRA 0 M/S:

    S=So+v.t+a.t²/2

    S=112,5+15.3-5.3²/2= 135 m

    *RESULTADO:

    -o espaço percorrido pelo motorista desatento é subtraído pelo espaço percorrido pelo motorista atento pra achsr a diferença.

    135-117,6= 17,4 metros de diferença

    -"como saber que os espaços percorridos a serem subtraídos seriam esses e não os primeiros que calculamos para cada motorista?"

    •as últimas equações (espaço enquanto a velocidade variava de 14 ou 15 m/s pra 0 m/s) incorporam os resultados desses primeiros espaços percorridos (espaço enquanto a velocidade variava de 0 m/s pra 14 ou 15 m/s) na posição inicial (So), por isso os espaços percorridos subtraídos entre si são esses mostrados ali em cima.


ID
2585503
Banca
IBFC
Órgão
SEE-MG
Ano
2015
Provas
Disciplina
Física
Assuntos

Um veículo de massa M, a uma velocidade V, ao ter seus freios totalmente acionados, necessita de uma distância d para parar totalmente (derrapar). Se este mesmo veículo estiver transportando uma carga de 0,5 M e estiver a uma velocidade 2 V, no mesmo tipo de pavimento, assinale a alternativa que indica a distância que ele necessitará para parar totalmente.

Alternativas
Comentários
  • GABARITO: C


    Se acharmos a proporção da quantidade de movimento (energia cinética) a gente achará a proporção da distância de frenagem nos dos casos. Fórmula da energia cinética: Ec=m.v²/2


    Vamos atribuir valor para ficar mais fácil.

    1 caso: M = 10 kg ---- V=10 m/s ----> Ec1 = 10.10²/2 ---> Ec1 = 500 Joules

    2 caso: M = 15 kg ---- V=20 m/s ----> Ec2 = 15.20²/2 ---> Ec2= 3000 Joules

    PORTANTO: 3000/500 = 6

  • Eu fiz utilizando o conceito do MRUV:

    Equação de Toricelli

    Vf²= V0² + 2.a.d

    Estado Inicial:

    Velocidade final é zero, pois o carro estará em repouso

    Sabe-se também que a aceleração é dada por: a=F/m

    0=V²+2.(-a).d

    V²=2.a.d

    V²=2.(F/M).d

    d=MV²/2F

    Para o segundo caso:

    0=(2V)²+2.(-a).d'

    4V²=2.F/1,5M.d'

    d'=6 MV²/2F

    d' = 6d LETRA C

  • Ec > Q


ID
2608963
Banca
Aeronáutica
Órgão
ITA
Ano
2017
Provas
Disciplina
Física
Assuntos

Quando precisar use os seguintes valores para as constantes: Constante da gravitação universal G = 7 x 10-11 m3/kg.s2. Aceleraçao da gravidade g = 10 m /s2. Velocidade do som no ar = 340 m/s. Raio da Terra R = 6400 km. Constante dos gases R = 8,3 J/mol.K. Indice adiabatico do ar y = CP/CV = 1,4. Massa molecular do ar Mar = 0,029 kg/mol. Permeabilidade magnetica do vacuo μ0 = 4π x 10-7 N/A2.

Pressão atmosferica 1,0 atm = 100 kPa. Massa específica da agua = 1 ,0 g/cm3

A partir de um mesmo ponto a uma certa altura do solo, uma partícula e lançada sequencialmente em três condições diferentes, mas sempre com a mesma velocidade inicial horizontal v0. O primeiro lançamento é feito no vácuo e o segundo, na atmosfera com ar em repouso. O terceiro é feito na atmosfera com ar em movimento cuja velocidade em relação ao solo e igual em módulo, direção e sentido à velocidade v0. Para os três lançamentos, designando-se respectivamente de t1 , t2 e t3 os tempos de queda da partícula e de v1, v2 e v3 os módulos de suas respectivas velocidades ao atingir o solo, assinale a alternativa correta.

Alternativas
Comentários
  • https://www.youtube.com/watch?v=AJUblhn052g


ID
2667661
Banca
INEP
Órgão
ENEM
Ano
2013
Provas
Disciplina
Física
Assuntos

Antes das lombadas eletrônicas, eram pintadas faixas nas ruas para controle da velocidade dos automóveis. A velocidade era estimada com o uso de binóculos e cronômetros. O policial utilizava a relação entre a distância percorrida e o tempo gasto, para determinar a velocidade de um veículo. Cronometrava-se o tempo que um veículo levava para percorrer a distância entre duas faixas fixas, cuja distância era conhecida. A lombada eletrônica é um sistema muito preciso, porque a tecnologia elimina erros do operador. A distância entre os sensores é de 2 metros, e o tempo é medido por um circuito eletrônico.


O tempo mínimo, em segundos, que o motorista deve gastar para passar pela lombada eletrônica, cujo limite é de 40 km/h, sem receber uma multa, é de

Alternativas
Comentários
  • omo o exercício quer a resposta em segundos:

    40km/h = 11,11m/s

    V = ΔS/Δt

    11,11 = 2/Δt

    11,11Δt = 2

    Δt = 2/11,11 ≈ 0,18s

  • Conversão de km/h para m/s: 40÷3,6=11,11 S=So+VoT 2=0+11,11.T 2/11,11=T 0,18=T
  • Como a distância é dada em metros, converte-se a velocidade de quilômetros por hora para metros por segundo:

    40Km/h * (5/18) = 100/9 m/s

    A fórmula para calcular o tempo decorrido, dada um distancia e velocidade contantes é Tempo = Distancia/Velocidade. Logo:

    T = 2 ÷ (100/9) = 0,18 s

    Resposta C

  • Distância entre os sensores: 2 metros

    Velocidade máxima: 40 km = 11 m/s

    Arredondando pra 10 m/s fica mais fácil resolver essa questão, porque não precisa fazer mais nenhum cálculo. Isso ajuda na hora da prova.

    Então, se em 1 segundo são percorridos 10 metros, isso quer dizer que em 2 metros (distancia dos sensores), são percorridos em 1/5 de segundo, ou seja: 0,2 segundos.

    A alternativa mais próxima é a C.

  • se a velocidade é de 40km/h :

    40.000m------------3600s

    2m-----------------xs

    simplificando e aplicando meio pelos extremos:

    36/200=x

    18/100=x

    x=0.18

    c

  • Errei pq esqueci de converter de Km/h pra m/s

  • corretissimo

  • Primeiramente, temos que converte de km/h para m/s

    Sendo assim fica 40 km/h---------- 11,11 m/s

    Colocando na formula do sorvete.

    S=So+Vo.T

    2=0+11,11.T

    2/11

    =0,18

    ALTERNATIVA C

  • Nível: Básico

    ΔV = ΔS÷Δt

    logo:

    Δt = ΔS÷ΔV

    Δt = 2÷(40÷3,6) - km/h -> m/s

    Δt = 2÷11,11

    .: Δt ≅ 0,18s (RESPOSTA: C)

  • tinha tudo pra acertar, só esqueci de converter @@ grr

  • converter de km/h para m/s dividi por 3,6

    V=40km/h

    D=2m

    como ele quer o tempo em segundos, converte km/h para m/s

    40/3,6 = 2/t

    3,6*2/40 = t

    0,18= t(segundos)

    alternativa C)

  • D= V . T

    (Deus vê tudo)

    2= 11.T

    T=2/11

    T= 0.18

  • passa tudo pra m/s ficara 40,000m em 3600s otimo !!!!

    agora uma regra de tres basica se ele faz 40000 em 3600

    quanto tempo levara se percorrer 2 metros assim NESSA MESMA VELOCIDADE DE 40KM/H

    40,000 ------ 3600

    2------- X RESOOLVE ISSO E CHEGARA NA RESPOSTA DE 0,18

  • 40 km/h para m/s:

    40000 m/s (40000 m em 3600s).

    40000 m — 3600s

    2 m — x

    40000x = 3600 • 2

    40000x = 7200

    x = 7200 / 40000

    x = 0,18

  • Galerinha, fórmula massa pra cinemática no Enem:

    V / 3,6 = d / t ~> 3,6 . D / V = t, assim: D × 3,6 / V = t;

  • um monte de blablabla e só importa a última linha kk

    enfim:

    converter para m/s: 40/3,6 = 400/36 = 100/9 m/s (deixa assim)

    v = s/t -> 100/9 = 2/t -> t= 0,18s

  • Petição pra ninguém ficar falando que é facil bl bl bla, me esforcei muito pra acertar kkkkkkk

  • errei por que multipliquei pra converter o km/h pra m/s ao inves de dividir

    agora escrevi uma folha inteira "km/h pra m/s é divisao"

    nao esqueço mais kkkk


ID
2681044
Banca
VUNESP
Órgão
UNESP
Ano
2018
Provas
Disciplina
Física
Assuntos

Um foguete lançador de satélites, partindo do repouso, atinge a velocidade de 5400 km/h após 50 segundos. Supondo que esse foguete se desloque em trajetória retilínea, sua aceleração escalar média é de

Alternativas
Comentários
  • Como a velocidade está em km/h e a questão deu as alternativas em m/s², então primeiramente converti a velocidade de km/h para m/s

     

    Lembrando que:

    km/h para m/s => divide por 3,6

    m/s para km/h => multiplica por 3,6

     

    5.400/3,6 = 1500m/s

     

    Agora basta usar a fórmula para encontrar a aceleração média:

    am = Δv/Δt

    am = 1500/50

    am = 30m/s²

     

    Gabarito: A

  • V = V0 + a.t

    1500 m/s = 0 + a. 50 s

    a = 1500 m/s / 50 s

    a = 30 m/s²

  • 54000/3,6 = 1500m/s² = velocidade

    em 50s = tempo

    Aceleração Escalar média= velocidade/tempo

    Vm= 1500/50

    Vm= 30m/s².

    LETRA A

    APMBB


ID
2741917
Banca
NUCEPE
Órgão
PC-PI
Ano
2018
Provas
Disciplina
Física
Assuntos

Um motorista viajando em seu carro a 90 km/h percebe uma árvore caída na pista a 60 m de distância. Ao acionar os freios, o carro desacelera a uma taxa constante de 5 m/s2. Com base nas informações dadas, é CORRETO afirmar que o carro:

Alternativas
Comentários
  • v² = v02 + 2aΔS  

     

    V²= 25m/s² - 2. 5 m/s. 60 m              ( transformei 90km/h em m/s  e  sinal negativo porque o veículo está desacelerando)

     

    V²= 625-600

    V²= 25                                   (raiz quadrada de 25)

    v= 5m/s                                 (transformando em km/h)

    v= 18km/h

     

    irá colidir com a árvore, a uma velocidade de 18 km/h.

     

    GABARITO E.

     

     

     

  • Dados da questão: 

    Vo = 90km/h ou 25m/s
    d = 60m 
    a = -5m/s² (movimento de desaceleração) 

    Vfinal²= Velocidade inicial² + 2 * aceleração * distância 
    Vfinal² = Vo + 2ad
    Vfinal²= 25² + 2*(-5)*60
    Vfinal² = 625 - 600
    Vfinal² = 25
    Vfinal = 5 m/s


    Ele irá colidir a 5m/s ou a 18km/h (passando de m/s para km/h basta multiplicar por 3,6 -> 5*3,6 = 18) 

     

    Gabarito E

  • Torricelli

  • a)     não colidirá com a árvore, parando a cerca de 1 m dela.

    Vamos tomar como base a velocidade final 0, ou seja, o momento em que o veículo estará completamente parado e analisar a distância percorrida pelo veículo. Caso este seja maior que 60, então teremos a conclusão que o veículo irá colidir com a árvore. Para isso utilizaremos a formula da velocidade de toricelli.

    V² = v0² + 2*a*Δs

    Velocidade: 90/3,6 = 25 m/s

    Aceleração: -5 m/s²

    0² = 25² + 2*(-5)*Δs

    0 = 625 -10*Δs

    10*Δs = 625

    Δs = 625/10 = 62,5 m

    Como falado anteriormente, o carro parará em 62,5, logo baterá na árvore.

    (ERRADO);

    b)     irá colidir com a árvore, a uma velocidade de 30 m/s.

    Usando a mesma formula anterior, saberemos qual velocidade ele atingirá a árvore.

    V² = v0² + 2*a*Δs

    V² = 25² + 2*(-5)*60

    V² = 625 – 600

    V² = 25

    V = 25 = 5 m/s

    (ERRADO);

    c)      não colidirá com a árvore, parando a cerca de 1 cm dela.

    MESMA JUSTIFICATIVA DA ASSERTIVA A).

    (ERRADO);

    d)     irá colidir com a árvore, a uma velocidade de 65 km/h.

    Tomando como base a resolução da assertiva B), então faremos apenas a conversão da velocidade final de m/s para km/h, ou seja, multiplicando por 3,6.

    V = 5 * 3,6 = 18 km/h

    Com este resultado podemos concluir que a assertiva E) é a correta.


ID
2741920
Banca
NUCEPE
Órgão
PC-PI
Ano
2018
Provas
Disciplina
Física
Assuntos

Três bolas idênticas, A, B e C, sujeitas apenas à ação da força gravitacional, são lançadas simultaneamente, com módulos de velocidade iguais, de uma mesma altura h do solo: uma verticalmente para cima ( A ), outra verticalmente para baixo ( B ) e a outra horizontalmente para a direita ( C ). Os módulos das velocidades com que as bolas atingem o solo será tal que:

Alternativas
Comentários
  • A questão afirma que as bolas foram lançadas e possuem módulos de velocidade iguais.  

    Se a bola C foi lançada horizontalmente, ela não deveria ter velocidade inicial no sentido vertical ( V0 = 0 ). Sendo assim, a velocidade de B (que é igual de A) ao solo deveria ser maior que a de C.

    Acredito que a alternativa A é a correta.

    REFORÇANDO MINHA DUVIDA: Acredito que lançamento de um corpo infere que existe uma velocidade maior que zero e para o caso do movimento vertical o lançamento não poderia ser equiparado como queda livre (caso das bolas A e B). No meu entender, a bola C, na vertical, caí com velocidade inicial zero e com apenas ação da gravidade (queda livre). O vetor de velocidade existe apenas na horizontal.

    Alguém pode fazer algum esclarecimento?

  • Um dado muito importante é que o problema diz que as bolas estão sujeitas apenas a força gravitacional;

    Alternativa (A)

    A bola foi lançada verticalmente, de uma altura h, para cima com a velocidade V, sendo assim atingirá uma determinada altura e depois voltará a cair com a mesma velocidade a qual foi lançada V, até a posição h que dai em diante será acrescentada a aceleração relativa a altura h;

    Alternativa (B)

    A bola é lançada com a mesma velocidade verticalmente para baixo, como a altura "h" nos dois casos é a mesma a velocidade com que a bola chega ao solo, Vb, será a mesma da alternativa (A),

    logo: Va=Vb

    Alternativa (C)

    Nessa bola a força aplicada e horizontalmente, assim de inicio temos apenas uma força V, entretanto ao chegar ao solo existira uma outra força produzida pela gravidade ao logo do percurso "h", assim o modulo da velocidade considerando as duas componentes será igual a das demais alternativas.


    Assim

    Va=Vb=Vc


  • GUILHERME DUARTE, PARA ESCLARECER A SUA DÚVIDA VOU CONTEXTUALIZAR A QUESTAO:

     

    PARA DESCOBRIR A VELOCIDADE COM A QUAL A BOLA  "C"  ATINGIRÁ O SOLO FAÇA ASSIM:

     

    DESENHE UM VETOR PARA DIREITA ( SIGNIFICANDO A VELOCIDADE NA HORIZONTAL)

     

    DESENHE UM VETOR PARA BAIXO  (SIGNIFICANDO A  VELOCIDADE NA VERTICAL, EMBORA (V0 = 0) GANHARÁ VALORES DEVIDO A ACELERAÇAO DA GRAVIDADE)

     

    AGORA EXTRAIA A VELOCIDADE RESULTANTE TENDO EM MENTE QUE A BOLA ATINGIRÁ O SOLO EM SENTIDO DIAGONAL

     

    VOU CONTEXTUALIZAR COM VALORES, POR EXEMPLO:

     

    BOLA "B"    ( V = V0 + AT) ESSA É A FORMULA DE CALCULAR

     

    BOLA "C"    (V* = V0* + AT*) O ASTERISCO SIGNIFICA AO QUADRADO. ESSA É A FORMULA DE CALCULAR, POIS É A SOMA DE DOIS VETORES

     

    PRONTO! AO COLOCAR VALORES, VOCE PERCEBERÁ QUE A VELOCIDADE DE  "A" "B" "C" SAO IGUAIS

     

    ESPERO TER AJUDADO!

     

  • Eu pensei que, como a bola que foi lançada verticalmente para baixo teria a soma da ação gravitacional, sua velocidade seria maior. Alguém poderia corrigir este meu raciocínio via in box? Agradeço!

  • Va² = V² + 2gh (velocidade resultante só na vertical); Vb² = V² + 2gh (velocidade só na vertical); Vc² = Vxc² + Vyc² (há velocidade resultante inclinada, pois há velocidades na horizontal e vertical) . Vxc = V e Vyc² = 2gh



    logo: Vc² = V² + 2gh


    portanto: Va=Vb=Vc

  • Irlan Gomes, seus cálculos estão corretos, contudo, não concordo com sua afirmação final, pois vejamos:


    Se Vc^2=V^2+(gt)^2, temos que isso é diferente de Vb, que é igual a V + gt, o que implica, segundo esses cálculos, que de fato a velocidade que C alcança o solo é inferior à A e B.

  • Conservação de Energia, sempre procurem pensar em métodos de energia antes de ir para a cinemática.

    Ki + Ui = Kf +Uf

    Ui, Uf e Ki são iguais para as três, logo Kf é igual para as três, como Kf = ½ m.vf² , a velocidade final das três é igual.

    Agora se ele tivesse perguntado sobre a velocidade vertical aí sim, va = vb > vc.

  • Paulo Afonso está corretíssimo. Os outros comentários não vale apena. Os tempos para as três bolas chegarem ao chão são diferente. Dessa forma igual é apenas a altura h. Usando equação de Torrichelii e comparar os resultados.

    Observação: A velocidade inicial de um corpo arremessado verticalmente é a mesma ao retornar então Va= Vb.

    Para calcular a velocidade de C= Vc

    Fazemos a resultante ( triangulo retângulo em que o quadrado da hipotenusa é igual ao quadrado dos catetos)

    velocidade Horizontal é Vc=Va=Vb mais

    velocidade na vertical ( equação de Torricheli V^2= (V inicial)^2 + 2 g. h

    Fica V^2= 2gh

    Calculando cada velocidade de cada bola separadamente verá o mesmo resultado de Vc.

  • Nesta questão vc pode resolver também por cálculo de Ep e Ec, pois considera-se conservativo.

    Vai perceber que todos irão dar mesmo valor.

    Você também pode atribuir valores às variáveis como por exemplo.

    => Todas as bolas são iguais, então vou considerar todas com massa de 10kg

    => Todas lançadas com mesma velocidade, então vou considerar todas v = 10m/s

    => g = 10m/s²

    => Lançados de uma determinada altuda h, vou considerar 10m do chão, que considerei como nível 0.

    Aí fica fácil. Vc calcula a Ep de B que nesse caso daria 1000.

    Faça o mesmo para A e C, porém será necessário fazer cálculo da Hmáx.

    É mais trabalhoso que as outras maneiras. Mas tb sai o resultado e vc acerta a questão que é o que importa!


ID
2750863
Banca
SEDUC - CE
Órgão
SEDUC-CE
Ano
2016
Provas
Disciplina
Física
Assuntos

Dois automóveis, I e II, inicialmente trafegam lado a lado em uma estrada reta. Em algum instante, o carro I aumenta sua velocidade e, simultaneamente, o outro começa uma frenagem. Assim, pode-se afirmar corretamente que

Alternativas
Comentários
  • GABARITO E.

     

    A aceleração dos dois carros são diferentes de zero. O carro l tem aceleração positiva e o carro ll aceleração negativa.

  • A frenagem também é uma aceleração, só que negativa.

  • O carro 1 acelera a>0

    O carro 2 desacelera a<0


ID
2761282
Banca
Quadrix
Órgão
SEDUC-GO
Ano
2018
Provas
Disciplina
Física
Assuntos

Suponha-se que a velocidade de uma partícula seja dada pela seguinte função: v = 50-30t + 20t2, onde t é dado em segundos e v em metros por segundo. Com base nessa informação, assinale a alternativa que apresenta a função da posição (x) em função do tempo, sabendo que a posição inicial é zero.

Alternativas
Comentários
  • Basta realizar a integral da função em relação ao Tempo.
    S = 50t - 30t²/2  + 20t³/3      Alternativa ~> C

  • f ( x ) =20 x^2 - 30 x + 50

     

    ∫ f ( x )d x= F ⋆ (x)

    Problema:

    ∫ (20 x^2 -30x+50)dx

    Aplique linearidade:

    = 20 ∫ x^2d x-30 ∫ xd x+50 ∫ 1d x

    Agora resolvendo:

    ∫ x^2d x

    Aplique a regra de energia:

    = x^3/3

    Aplique a regra de energia com 

    = x^2/2

    Agora resolvendo:

    ∫ 1d x

    Aplicar regra constante:

    = x

    Conecte integrais resolvidas:

    20 ∫ x^2d x-30 ∫ xd x+50 ∫ 1d x

    = 20 x^3/3- 15 x^2 + 50 x

    O problema está resolvido:

    ∫ (20 x^2 -30x+50)dx

    = 20 x^3 /3- 15 x^2 + 50 x

  • As questões de Física deveriam ser respondidas pelos professores e em vídeo. Não fica mt útil esctudar respondendo questões de física sem a respota explicativa dos professores.

  • Não entendi bulhufas...

  • Quem não é do ramo da física/engenharia tem dificuldades nesse tipo de questão, e é normal, fiquem tranquilos quem estiver estudando para área policial esse tipo de questão nãoirá cair!!

    Agora para quem for curioso, basta saber:

     

    função velocidade, quando Integrada vira a função posição.

    função posição, quando derivada vira a função velocidade.

     

    Em curta síntese, a derivada é o contrário da integral.

  • a equação da velocidade é a derivada da equação da distancia e a equação da aceleração é a derivada da equação da velocidade.

    Para achar a derivada multiplica o expoente pelo numero na frente do t e o número que fica no expoente do t, é o expoente inicial menos 1

    ex: 50t - 15t² + 20t³/3 

    1x50t^(1-1) - 2x15t^(2-1) + 3x20/3t^(3-1)

    50-30t+20t^2

  • f ( x ) =20 x^2 - 30 x + 50

     

    ∫ f ( x )d x= F ⋆ (x)

    Problema:

    ∫ (20 x^2 -30x+50)dx

    Aplique linearidade:

    = 20 ∫ x^2d x-30 ∫ xd x+50 ∫ 1d x

    Agora resolvendo:

    ∫ x^2d x

    Aplique a regra de energia:

    = x^3/3

    Aplique a regra de energia com 

    = x^2/2

    Agora resolvendo:

    ∫ 1d x

    Aplicar regra constante:

    = x

    Conecte integrais resolvidas:

    20 ∫ x^2d x-30 ∫ xd x+50 ∫ 1d x

    = 20 x^3/3- 15 x^2 + 50 x

    O problema está resolvido:

    ∫ (20 x^2 -30x+50)dx

    = 20 x^3 /3- 15 x^2 + 50 x

  • A letra B é a equação da aceleração instantânea.

  • meu Deus comecei agora e já penso em desistir .
  • Tem outro jeito de resolver não? não sendo por integral...

  • v = ds/dt => ds = v.dt => s = Integral ( v.dt) = > s = Integral (50-30t+20t^2)


ID
2773345
Banca
UFU-MG
Órgão
UFU-MG
Ano
2017
Provas
Disciplina
Física
Assuntos

Ao se projetar uma rodovia e seu sistema de sinalização, é preciso considerar variáveis que podem interferir na distância mínima necessária para um veículo parar, por exemplo. Considere uma situação em que um carro trafega a uma velocidade constante por uma via plana e horizontal, com determinado coeficiente de atrito estático e dinâmico e que, a partir de um determinado ponto, aciona os freios, desacelerando uniformemente até parar, sem que, para isso, tenha havido deslizamento dos pneus do veículo. Desconsidere as perdas pelas resistência do ar e o atrito entre os componentes mecânicos do veículo. A respeito da distância mínima de frenagem, nas situações descritas, são feitas as seguintes afirmações:

I. Ela aumenta proporcionalmente à massa do carro.
II. Ela é inversamente proporcional ao coeficiente de atrito estático.
III. Ela não se relaciona com a aceleração da gravidade local.
IV. Ela é diretamente proporcional ao quadrado da velocidade inicial do carro.

Assinale a alternativa que apresenta apenas afirmativas corretas.

Alternativas
Comentários
  • I. Ela aumenta proporcionalmente à massa do carro.

    Errado. A força de atrito estático é N*u = mg * u, onde u é o coeficiente de atrito estático. Esta força equivale à resultante horizontal, ou seja F = m*a. Uma vez que a resultante é a força de atrito estático, temos ma= mgu. Cancela a massa dos dois lados, fica a = gu. Sendo assinto, a aceleração resultante só depende da aceleração gravitacional local e do coeficiente de atrito estático.

    III. Ela não se relaciona com a aceleração da gravidade local.

    Uma vez que a distância mínima de frenagem é ΔS, tal que V2 = V0 - 2a ΔS, percebemos de imediato que a e ΔS são inversamente proporcionais. Pegando a fórmula em negrito la de cima, percebemos que ΔS e g são inversamente proporcionais.


ID
2848021
Banca
UFPR
Órgão
PM-PR
Ano
2018
Provas
Disciplina
Física
Assuntos

Um objeto move-se numa pista retilínea, descrevendo um movimento retilíneo uniformemente variado, quando observado por um sistema de referência inercial. A posição desse objeto é descrita pela equação x(t) = 5 – 6t + 3t2 , onde x é medido em metros e t em segundos. Sabe-se que a massa do objeto é fixa e vale m = 600 g. Tendo em vista essas informações, considere as seguintes afirmativas:

1. A posição inicial do objeto vale 5 m.
2. A força agindo sobre o objeto durante o movimento vale, em módulo, F = 3,6 N.
3. O objeto tem velocidade nula em t = 1 s.
4. No intervalo de t = 0 a t = 3 s, o objeto tem deslocamento total nulo.

Assinale a alternativa correta.

Alternativas
Comentários
  • D

  • Ele deu a função posição do objeto: x(t)5-6t+3t²

    -------------------------------------------------------------------------------------------------------------

    Para encontrar a velocidade, derivamos a posição em função do tempo, temos:

    V=ds/dt

    V=d(5-6t+3t²)/dt

    V=(-6+6t)m/s

    --------------------------------------------------------------------------------------------------------------

    Para encontrar a aceleração, derivamos a velocidade em função do tempo, temos:

    a=dv/dt

    a=d(-6+6t)/dt

    a=6m/s²

    -------------------------------------------------------------------------------------------------------------

    Agora:

    I - Posição inicial, t=0s

    x(0)=5-6t+3t² => 5-6(0)+3(0)² => x(0)=5m VERDADEIRO

    II - Força sobre objeto

    F=m.a => 0,6kg.6m/s² => F=3,6N VERDADEIRO

    III - Velocidade em t=1s

    v(1)=-6+6(1) => v(1)=0 VERDADEIRO

    IV - Deslocamento total

    x(0)=5-6t+3t² => 5-6(0)+3(0)² => x(0)=5m

    x(3)=5-6t+3t² => 5-6(3)+3(3)² => x(3)=14m

    Saiu da posição 5m e chegou a 14m em 3s, logo deslocamento foi de 9m. FALSO


ID
2852224
Banca
Aeronáutica
Órgão
EEAR
Ano
2018
Provas
Disciplina
Física
Assuntos

Em um trecho de uma rodovia foram instalados conjuntos de cronômetros digitais. Cada conjunto é formado de dois sensores distantes 2 km entre si que registram o horário (hora, minuto e segundo) em que um mesmo veículo, deslocando-se no mesmo sentido, passa por eles. Em um trecho da rodovia no qual a velocidade média permitida é de 100 km/h, um carro a 120 km/h atinge o primeiro de um desses conjuntos exatamente às 15h00min00s. O horário em que esse veículo deve passar pelo segundo sensor de forma a percorrer esse trecho da rodovia exatamente com velocidade média igual a 100 km/h é

Alternativas
Comentários
  • d = 2km

    "de forma a percorrer esse trecho da rodovia exatamente com velocidade média igual a 100 km/h"


    V = d/t = t = d/v

    t = 2/100 = 0,02h


    3600 x 0,02 = 72 segundos


    72 segundos = 1 minuto e 12 segundos


    Alternativa A) 15h01min12s

  • De onde tu tirou esse 3600?

  • temos:

    d = 2km

    v = 100km/h

    eu achei melhor transformar tudo para metro e m/s

    ficando:

    d = 2000m

    v = 100 km/h / 3,6 = 27,7 m/s

    então...

    t = d/v

    t = 2000/27,7

    t = 72,2 segundos

    72 segundos = 1 minuto e 12 segundos

    letra: A

  • Denison, 1h tem 3600 segundos então 3600x0,02= 72 segundos

  • Fiz por regra de três!

    100 ----- 60

    2 ------ x

    100x = 120

    x = 120/100

    x = 1,2 minutos

    1,2 minutos = 72 segundos

    15h00min0s + 72 s = 15h01min12s

  • Usei a equação horária do M.U:

    S = S(inicial) + v x t

    2 = 4 + 100 x t

    100t = 4 - 2

    t = 2/100

    t= 0,02 h

    transformar hora em minuto e em seguida somar ao tempo que ele atinge o primeiro conjunto:

    basta multiplicar por 60

    0,02 x 60 = 1,20

    ou seja, 1 min e 20 segundos

    15h + 1,20 mim = 15h01min12s

    Letra A

    Obs.: usei o 4 no S0, porque somei 2km mais os próximos 2km que ele percorreria para passar pelo 2º sensor

    :*

  • Denison, 1 hora = 60 minutos = 3600 segundos

  • Vm = Δs/ Δt

    100 = 2 / Δt

    Δt = 2 / 100

    Δt = 0,02h

    0,02h = 72s = 1m 12s

    Ou seja, 15:01:12

  • tá safe

  • Alguém pode me explicar pq eu ñ poderia abrir o Delta T = t-t' , pq eu falei que Delta t era tempo final que era t - o inicial que era 15 e fui resolvendo e acabei encontrando 15,02 h.

  • Lorena Gouveia, com esse resultado você pode fazer uma regra de três e descobrir quanto é 0,02 horas em minutos, assim você acha a resposta
  • Atenção! Nessa questão há uma pegadinha. Ele fala velocidade do avião(120km/h), porém manda nós considerarmos a velocidade=100km/h no trecho de 2km.

    d=2km

    v=∆s/∆t

    ∆t=∆s/v

    ∆t=2/100

    ∆t=0,02h -> 3600.0,02=72s = 1min e 12s

    R:15h 1min e 12s


ID
2853583
Banca
UECE-CEV
Órgão
UECE
Ano
2018
Provas
Disciplina
Física
Assuntos

Considere um veículo de massa constante que se desloca em linha reta. Este veículo tem seu momento linear dado por p = 4t , onde t é o tempo e a constante multiplicativa 4 tem a unidade de medida apropriada. Assim, é correto afirmar que

Alternativas
Comentários
  • GABARITO B.

     

    O  momento linear é definido pela quantidade de movimento, a qual é definido pelo produto da massa e sua velocidade.

    Q= m.v

     

    Como o carro possui quantidade de movimento e sua massa constante então irei concluir que ele está em uma certa velocidade. Essa velocidade adquiriu com o passar do tempo, ou seja esse tempo multiplicando por 4. A aceleração é constante.

     

    Conforme a fórmula da energia cinética concluimos que se a velocidade do veiculo está aumentando, então a energia cinética está aumentando também, ou seja crescente.

    Ec= m.v²/2

     

     

     

  • Se a velocidade varia 4.t ,logo a cada intervalo de tempo

    há uma constância de aceleração, sendo mruv.

     

    Alternativa: B

  • Aplicando as fórmulas de quantida de movimento e energia cinética, percebe-se que a velocidade e a energia cinética n podem ser constantes, pois se ocorrer variação do tempo, o valor será alterado ( ou seja, dependem diretamente do tempo ). Além disso, a energia cinética n pode ser decrescente por ter o tempo t elevado ao quadrado, determinando uma crescimento ( progressão ) da energia cinética. A única alternativa coerente é sobre a aceleração ser constante! Espero que meu raciocínio esteja certo e que tenham entendido.